Propriétés du produit

Combustion 1 8. Lignes directrices pour résoudre les problèmes et. Problèmes à résoudre de manière autonome

Combustion 1 8. Lignes directrices pour résoudre les problèmes et.  Problèmes à résoudre de manière autonome

Saratov 2010

Compilé par : R.P. Volkov, enseignant à l'établissement d'enseignement fédéral d'enseignement professionnel secondaire « SGPPC du nom de Yu. A. Gagarin »

Réviseur interne : O.G. Stegalkina – enseignant

FGOU SPO "SGPPC du nom de Yu.A. Gagarine"

Instructions méthodologiques pour résoudre des problèmes et effectuer un travail indépendant dans le cours « Fondements physico-chimiques du développement et de l'arrêt de la combustion en cas d'incendie » pour les étudiants de toutes formes d'études dans la spécialité « Sécurité incendie ».

Les lignes directrices présentent des exemples de résolution de problèmes typiques dans la section « Principes fondamentaux des processus de combustion. Bilan matériel et thermique des procédés de combustion" discipline "Fondements physico-chimiques du développement et de l'arrêt de la combustion en cas d'incendie" ; Des options de tâches permettant de résoudre des problèmes de manière indépendante sont proposées.

Imprimé dans l'imprimerie de l'établissement fédéral d'enseignement secondaire "SGPPC du nom de Yu.A. Gagarine"

INTRODUCTION

Instructions méthodologiques pour résoudre les problèmes et effectuer un travail indépendant dans la section « Fondamentaux des processus de combustion. Bilan Matière et Thermique des Procédés de Combustion » discipline « Fondements physico-chimiques du développement et de l'arrêt de la combustion en cas d'incendie » sont destinés à la formation des ingénieurs en sécurité incendie dans le cadre du programme de travail de la discipline « Fondements physico-chimiques du développement et cessation de la combustion en cas d'incendie » dans la spécialité 280104.

Les lignes directrices pour résoudre les problèmes sont élaborées dans le plein respect des normes éducatives de l'État pour l'enseignement professionnel supérieur, en tenant compte des caractéristiques des activités professionnelles des employés des pompiers de l'État. Les travaux visent à consolider le cours théorique et les méthodes de calculs pratiques dans cette section de la discipline. Des instructions méthodologiques aideront les étudiants à maîtriser la matière de la discipline étudiée, nécessaire au bon travail d'un ingénieur en sécurité incendie dans n'importe quel domaine de son activité.

Les instructions méthodologiques comprennent : de brefs principes théoriques, des dispositions générales pour calculer le bilan matériel et thermique des processus de combustion de substances gazeuses et condensées, la nature de la lueur de la flamme, la température de combustion, ainsi qu'un grand nombre d'exemples de résolution de problèmes typiques et de référence informations nécessaires à la résolution des problèmes.

La structure et le contenu des instructions méthodologiques pour résoudre des problèmes offrent aux étudiants la possibilité de pratiquer de manière indépendante le matériel dans chaque section de la discipline.

Lorsqu'on commence à étudier le cours, il faut imaginer que la base de tous les phénomènes se produisant lors d'un incendie est le processus de combustion. La connaissance de l'essence de ce phénomène, des lois de la combustion, des mécanismes et des méthodes d'arrêt est nécessaire au bon travail d'un ingénieur en sécurité incendie dans n'importe quel domaine de son activité.

1. Écrire des formules développées, établir des équations de réaction pour la combustion de substances inflammables dans l'air et calculer des coefficients stoechiométriques.

1.1. amylbenzène, acide abiétique, allylamine ;

1.2. amyldiphényle, acide adipique, isothiocyanate d'allyle ;

1.3. Amylène, acide acrylique, alnaft ;

1.4. amylnaphtalène, acétate d'allyle, altax;

1.5. amyltoluène, diacétate d'allylidène, amylamine;

1.6. anthracène, caproate d'allyle, nitrate d'amyle ;

1.7. acénaphtène, alcool allylique, nitrite d'amyle ;

1.8. acétylène, acétate d'amyle, sulfure d'amyle ;

1.9. le benzène, le butyrate d'amyle, l'amyle trichlorosilane ;

1.10. le butylbenzène, l'amylxylyl éther, l'amylchloronaphtalène ;

1.11. le butylcyclohexane, le laurate d'amyle, l'aminalone ;

1.12. butylcyclopétane, amylméthylcétone, colorant aminoazoïque ;

1.13. hexadécane, amyloléate, acide aminocaproïque ;

1.14. hexane, salicylate d'amyle, acide aminopélargonique ;

1.15. hexylcyclopentane, stéarate d'amyle, aminocyclohexane ;

1.16. heptadécane, amylphénylméthyléther, ampicilline ;

1.17. heptane, éther amphénylique, angine de poitrine;

1.18. décane, formiate d'amyle, aniline ;

1.19. le diamylbenzène, l'anisole, l'anthrimide ;

1.20. diamylnaphtalène, acétal, atophane ;

1.21. divinylacétylène, acétaldéhyde, acéclidine ;

1.22. dihydrocyclopentadiène, acétylacétone, acétanilide ;

1.23. diisobutylène, acide acétisalicylique, chlorure d'acétyle ;

1.24. diisopropylbenzène, citrate d'acétyltributyle, acétoacétanilide;

1.25. diméthylènecyclobutane, acétométhoxane, acétonitrile ;

1.26. ditolylméthane, acétone, acétoxime ;

1.27. biphényle, acétonylacétone, acétoéthylamide;

1.28. diphénylméthane, alcool acétopropylique, benzamide ;

1.29. diéthylcyclohexane, éther acétoacétique, benzyldiéthylamine ;

1.30. dodécane, acétophénone, benzylthiol ;

1.31. l'isobutylbenzène, le benzaldéhyde, le chlorure de benzyle ;

1.32. l'isobutylcyclohexane, la benzanthrone, le cyanure de benzyle ;

1.33. l'isooctane, le benzhydrol, le benzimidazole;

1.34. isopentane, acétate de benzyle, benzoate de sodium ;

1.35. l'isoprène, le benzoate de benzyle, le chlorure de benzoyle ;

1.36. l'isopropénylbenzène, le salicylate de benzyle, la benzoxazolone ;

1.37. l'isopropylacétylène, le benzyl cellosolve, le benzène sulfazide ;

1.38. le méthylcyclohexane, l'éther benzyléthylique, le benzène sulfamide ;

1.39. le méthylcyclopentane, l'acide benzylsuccinique, l'acide benzènesulfonique ;

1.40. octyltoluène, acétate de méthoxybutyle, benzonitrile.

2. Écrire des formules développées et déterminer la combustion de quelle substance inflammable libérera le plus grand nombre de moles de produits de combustion ?

2.1. la benzophénone et l'acide benzophénonetracarboxylique ;

2.2. bornéol et butanal;

2.3. l'acide butanoïque et l'acétate de butyle ;

2.4. ricinoléate de butylacétyle et butylacétoacétate

2.5. le sébacate de butyle benzyle et le benzoate de butyle ;

2.6. le butyrate de butyle et l'éther butylvinylique;

2.7. le butyle glycol et l'acétate de butyle glycol ;

2.8. l'éther de butylglycide et l'adipate de butyldiéthyle ;

2.9. l'isovalérate de butyle et le capronate de butyle ;

2.10. le butylcarbitol et le lactate de butyle ;

2.11. le laurate de butyle et le méthacrylate de butyle ;

2.12. la butylméthylcétone et l'oléate de butyle;

2.13. le propionate de butyle et le ricinooléate de butyle ;

2.14. le stéarate de butyle et l'éther de butylphényle ;

2.15. le formiate de butyle et le butyléthylacétaldéhyde ;

2.16. la butyléthylcétone et l'éther butyléthylique;

2.17. l'acide valérique et le valéraldéhyde ;

2.18. vanilline et acétate de vétivéryle;

2.19. alcool de vétiver et acétate de vétinyle ;

2.20. la vétinone et l'éther vinylallylique ;

2.21. l'acétate de vinyle et le butyrate de vinyle ;

2.22. l'éther vinylisobutylique et l'éther vinylisotylique;

2.23. l'éther vinylisopropylique et le crotonate de vinyle ;

2.24. la vinylméthylcétone et le méthacrylate de vinyloxyéthyle ;

2.25. l'éther vinyloctadécylique et le propionate de vinyle ;

2.26. l'éther vinyltriméthylnonylique et l'éther vinyléthylique;

2.27. éther vinyléthylique et acide tartrique;

2.28. vitamine A (acétate) et vitamine C ;

2.29. acide gallique et hexanal;

2.30. l'acide hexanoïque et l'acétate d'hexyle ;

2.31. l'hexylbutyrate et l'hexyldiéthylhexahydrophtalate ;

2.32. le méthacrylate d'hexyle et l'hexylméthylcétone ;

2.33. l'alcool hexylique et le propionate d'hexyle ;

2.34. le formiate d'hexyle et l'hexyl cellosolve ;

2.35. l'héliotropine et l'alcool heptadécylique ;

2.36. heptanal et heptylacétate;

2.37. l'heptylbutyrate et l'heptyldiphénylcétone ;

2.38. l'heptylisobutylcétone et l'heptylméthylcétone;

2.39. l'alcool heptylique et le propionate d'heptyle ;

2.40. formiate d'heptyle et hydroquinone.


SOLUTION.

1. Nous compilons des équations pour les réactions de combustion de mélanges de gaz combustibles dans l'air :

C 2 H 2 + 2,5 (O 2 + 3,76 N 2) = 2 CO 2 + H 2 O + 2,5*3,76N2 ,

C 3 H 8 + 5 (O 2 + 3,76 N 2) = 3 CO 2 + 4 H 2 O + 5*3,76N2.

2. Calculons les volumes théoriques d'air et de produits de combustion lors d'une combustion complète de 1 m3 de mélange gazeux (formules 8 et 15) :

3. Calculons les volumes réels d'air et de produits de combustion en tenant compte d'un excès d'air de 40 % (α = 1,4).

4. Étant donné que le volume du mélange combustible était de 10 m3, les volumes réels d'air et de produits de combustion seront respectivement de 176,7 et 192,9 m3.

RÉPONDRE: La combustion de 10 m 3 d'un mélange gazeux complexe nécessite 176,7 m 3 d'air, ce qui produit 192,9 m 3 de produits de combustion.

EXEMPLE:Déterminer les volumes d'air et de produits de combustion lors de la combustion de 2 kg de substance combustible ayant la composition élémentaire : C = 50 % ; N = 10 % ; N = 10 % ; cendres = 12 % ; humidité = 18%. Supposons que l'air et les produits de combustion se trouvent dans des conditions normales.

SOLUTION:

1. Pour résoudre le problème, nous utiliserons les formules (9) et (16).


Lorsque 2 kg de substance combustible sont brûlés, 14,34 et 16,14 m3 d'air et de produits de combustion se forment respectivement.

RÉPONDRE: Lorsque 2 kg de substance combustible sont brûlés, 14,34 m3 d'air sont consommés et 16,14 m3 de produits de combustion se forment.

TÂCHES POUR UNE SOLUTION INDÉPENDANTE

1. Déterminer le volume d'air nécessaire à la combustion de 50 m 3 d'acétylène à α = 1, 7.

2. Déterminer les volumes d'air, les produits de combustion et le pourcentage de produits de combustion contenus dans 2 m 3 d'éthane. Prenons la température des produits de combustion de 1 200 K, la pression de 101,3 kPa, l'excès d'air α=1,2.

3. Déterminer le volume d'air nécessaire à la combustion de 15 m 3 de butane à une température de 10°C et une pression de 750 mm Hg. Art., si la combustion se produit avec un coefficient d'excès d'air égal à 1,4 (α = 1,4).

6. Calculez la quantité d'amylbenzène qui peut brûler dans une pièce fermée d'un volume de 200 m 3 si la combustion s'arrête à une teneur en oxygène résiduel de 12 %. La température ambiante initiale est de 24 °C et la pression est de 98 kPa.

7. Déterminez combien d'acétate de butyle peut brûler dans une pièce d'un volume de 200 m 3 si sa combustion s'arrête à une teneur en oxygène de 13,8 % (conditions normales).

8. Déterminez les volumes de produits de combustion et d'air lors de la combustion de 7 kg d'hexane. Le processus de combustion s'est déroulé à une température de 33 o C et à une pression de 730 mm. art. Art. La température des produits de combustion est estimée à 1300 K.

9. Déterminez les volumes de produits de combustion et d'air lors de la combustion de 11 kg d'acétone. Le processus de combustion s'est déroulé à une température de 30 o C et à une pression de 720 mm Hg. Art. La température des produits de combustion est estimée à 1300 K.

10. Déterminez le volume de produits de combustion et d'air lors de la combustion de 17 kg de toluène. Le processus de combustion s'est déroulé à une température de 30 o C et à une pression de 745 mm Hg. Art. La température des produits de combustion est estimée à 1 100 K.

11. Calculer le volume d'air et le volume de produits de combustion lors de la combustion complète de 6 kg de cellulose, constitués de 80 % de carbone, 13 % d'hydrogène et 7 % d'oxygène, si la combustion se produit à une température de 25 o C et une pression de 95 kPa . Le coefficient d'excès d'air est de 1,4.

12. Déterminer le volume d'air nécessaire à la combustion de 6 kg d'éther diéthylique à une température de 15 o C et une pression de 750 mm Hg. Art. Le coefficient d'excès d'air était de 1,3.

13. Déterminez la quantité de benzène brûlée dans une pièce fermée d'un volume de 180 m3, si l'on sait que sa combustion s'est arrêtée lorsque la teneur en oxygène de l'air était de 14,6 %. La température avant l'incendie était de 19 o C et la pression était de 100 kPa.

15. Déterminer le coefficient d'excès d'air si 212 m 3 d'air sont consommés pour la combustion de 8 kg d'acétate d'éthyle à une température de 25 o C et une pression de 760 mm Hg. Art.

16. Calculez le coefficient d'excès d'air et le pourcentage de dioxyde de carbone dans les produits de combustion si 70 m3 d'air sont consommés pour la combustion complète de 4 kg d'éther éthylpropylique (C5H12O) à une température de 22 o C et une pression de 92 kPa.

17. 3 kg d'acroléine brûlent à une température de 21 o C et une pression de 98 kPa. Calculez le volume d'air converti en produits de combustion et le pourcentage d'eau qu'ils contiennent si la combustion se produit avec un excès d'air (le coefficient d'excès d'air est de 1,2).

20. Calculez le volume d'un mélange gazeux composé de 45 % de butane, 30 % de méthane, 20 % d'acétylène et 5 % d'oxygène, si 80 m 3 d'air sont consommés pour sa combustion dans des conditions normales. Le coefficient d'excès d'air est de 1,6.

22. Calculer le volume d'air et le volume de produits de combustion lors de la combustion complète de 7 m 3 d'un mélange gazeux composé de 57 % d'hydrogène, 18 % de monoxyde de carbone et 25 % de méthane, si la combustion se produit avec un excès d'air (le coefficient d'excès d'air est de 1,3 ).

23. Calculer le volume d'air et le volume de produits de combustion lors de la combustion complète de 6 kg de cérésine, composé de 80 % de carbone, 15 % d'hydrogène et 5 % d'oxygène, si la combustion se produit à une température de 25 o C et une pression de 95 kPa . Le coefficient d'excès d'air est de 1,5.

25. Déterminer le volume et la composition des produits de combustion (en %) du mélange gazeux (tableau 4), si la combustion se produit avec un coefficient d'excès d'air α (voir tableau 4).

Tableau 4

Composition du mélange, % Numéro de travail
25.1 25.2 25.3 25.4 25.5 25.6 25.7 25.8 25.9 25.10
Monoxyde de carbone - - - - - -
Hydrogène - - - - - - -
Méthane - - - - - -
Éthane - - - - - --
Propane - - - - - - - - -
Butane - - - - - - - -
Éthylène - - - - -
Propène - - - - - - - -
Acétylène - - - - - - -
Gaz carbonique - -
Azote - - - -
Oxygène - - -
α 1,2 1,3 1,1 1,2 1.2 1,2 1,4 1,1 1,3

Tableau 5

Numéro de travail Substance Composition élémentaire de la substance à C Masse de substance, kg
C H Ô S W cendre
26.1 Cérésine
26.2 Charbon
26.3 Bois
26.4 Essence
26.5 Huile
26.6 Essence
26.7 Kérosène
26. 8 Schistes bitumineux
26.9 Charbon
26.10 Anthracite 0,2 5,8

27. Déterminez la nature de la lueur de la flamme d’éthylbenzène.

28. Déterminez la nature de la lueur de la flamme de l’acide acétique.

29. Déterminez la nature de la lueur de la flamme de l'hexane.

30. Déterminez la nature de la lueur de la flamme de l’alcool amylique.

31. Déterminez la nature de la lueur d’une flamme de butane.

32 . Déterminez la nature de la lueur d’une flamme de benzène.

CALCUL DE LA TEMPÉRATURE DE COMBUSTION

La température de combustion fait référence à la température maximale à laquelle les produits de combustion sont chauffés. En ingénierie et en lutte contre les incendies, on distingue les températures de combustion théoriques, calorimétriques, adiabatiques et réelles.

Température de combustion théorique est la température à laquelle la chaleur libérée par la combustion d'un mélange de composition stoechiométrique est dépensée pour le chauffage et la dissociation des produits de combustion. En pratique, la dissociation des produits de combustion commence à des températures supérieures à 2 000 K.

R.

Température de combustion calorimétrique– c'est la température qui est atteinte lors de la combustion d'un mélange combustible stoechiométrique avec une température initiale de 273 K et en l'absence de pertes dans l'environnement.

Température de combustion adiabatique est la température de combustion complète de mélanges de toute composition en l'absence de pertes de chaleur dans l'environnement.

Température de combustion réelle est la température de combustion atteinte lors d'un incendie réel. Elle est bien inférieure aux valeurs théoriques, calorimétriques et adiabatiques, car dans des conditions réelles, jusqu'à 40 % de la chaleur de combustion est généralement perdue par rayonnement, sous-combustion, chauffage de l'excès d'air, etc.

La détermination expérimentale de la température de combustion pour la plupart des substances inflammables présente des difficultés importantes, notamment pour les liquides et les solides. Cependant, dans un certain nombre de cas, la théorie permet de calculer la température de combustion de substances avec une précision suffisante pour la pratique, sur la seule base de la connaissance de leur formule chimique, de la composition du mélange combustible initial et des produits de combustion.

Dans le cas général, la dépendance suivante est utilisée pour les calculs (approximative, puisque С р =f(T)) :

Q pg = V pg *C p *T g,

où Q pg est l'enthalpie des produits de combustion ;

V pg - quantité de produits de combustion, m 3 /kg ;

C r - capacité thermique volumétrique moyenne du mélange de produits de combustion dans la plage de température de T 0 à T g, kJ/(m 3 * K) ;

Tg - température de combustion, K.

L'enthalpie des produits de combustion est déterminée à partir de équations du bilan thermique:

Q pg = Q H + Q out – Q sueur, (24)

Q sueur = Q et + Q sous + Q dis Avec , (25)

FAI Q– chaleur d'évaporation ;

Q sueur– les pertes de chaleur dues à radiation, sous-combustion Et dissociation produits de combustion.

Selon le type de déperdition thermique pris en compte dans la zone de combustion (rayonnement, sous-combustion, dissociation des produits de combustion), l'une ou l'autre température est calculée.

Lors de la combustion cinétique de mélanges gaz-vapeur-air, la perte de chaleur de la zone de combustion est négligeable. Par conséquent, pour ces mélanges, la température de combustion réelle est proche de l'adiabatique, qui est utilisée dans les calculs techniques d'incendie.

Il est très difficile de déterminer la capacité calorifique moyenne d'un mélange de produits de combustion. Approximativement, l'enthalpie d'un mélange de produits de combustion peut être exprimée comme la somme des enthalpies de ses composants :

Q pg =Σ (V pg) je (C p) je*Tg, (26)

å ×
ni l'un ni l'autre

(V pg) je– la quantité du ième composant des produits de combustion ;

Sp est la capacité thermique volumétrique moyenne du i-ème composant à Tg et une pression constante ;

Tg– la température de combustion.

Lors du calcul de la température de combustion, utilisez la valeur Q n(pouvoir calorifique inférieur), car à la température de combustion, l'eau est à l'état gazeux.

Les valeurs de la chaleur inférieure de combustion d'une substance (effet thermique d'une réaction chimique) sont données dans la littérature de référence, et peuvent également être calculées à partir du corollaire de la loi de Hess :

Q n =(Σ ΔН i *n i) cont -(ΣΔН i *n i) out, où (27)

ΔН i – chaleur de formation de la ième substance,

n je– nombre de moles de la ième substance.

Selon corollaire de la loi de Hess L'effet thermique d'une réaction chimique est égal à la différence entre les sommes des chaleurs de formation des produits de réaction et les chaleurs de formation des substances de départ. Rappelons du cours de chimie que la chaleur de formation des substances simples (oxygène, azote, etc.) est nulle.

Par exemple, calculons la chaleur de combustion (effet thermique) de l'éthane :

C2H6 + 3,5*(O 2 + 3,76N 2) = 2 CO 2 + 3 H 2 O + 3,76 *3,5N2.

Le pouvoir calorifique le plus bas, selon le corollaire de Hess, est égal à :

Q n =ΔH CO 2 *n CO 2 +Δ H H 2 O * n H 2 O - ΔH C 2 H 6 *n C 2 H 6 (28)

En remplaçant les valeurs de la chaleur de formation de CO 2, H 2 O, C 2 H 6 à partir des données de référence, la chaleur inférieure de combustion de l'éthane est déterminée.

Lors de la combustion d'un mélange de substances individuelles, la chaleur de combustion de chaque composant est d'abord déterminée, puis résumée en tenant compte du pourcentage de chaque substance combustible dans le mélange :

Si le carburant est une substance complexe et que sa composition élémentaire est spécifiée en pourcentage en masse, alors la formule de Mendeleïev est utilisée pour calculer la chaleur de combustion :

Q n c m =339,4*C+1257*H-108,9(O-N-S)-25(9*H+W), kJ/kg (30)

où C,H,O,N,S est la teneur en pourcentage d'un élément donné dans une substance combustible ;

W– teneur en humidité en masse. %.

Pour calculer la température de combustion, nous établissons une équation de bilan thermique, en supposant que la chaleur dégagée par la combustion réchauffe les produits de combustion à partir de la température initiale T 0 jusqu'à la température T g.:

Q n (1-η)=Σс rpg i *V pg i (T g -T 0)

Où η coefficient de perte de chaleur (la proportion de perte de chaleur due au rayonnement, ainsi qu'à une combustion incomplète) ;

avec RPG je capacité calorifique du ième produit de combustion à pression constante, kJ/molK ;

V pg i – volume du i-ème produit de combustion, m3.

Calcul du volume de produits de combustion ( CO 2, H 2 O, SO 2, N 2) s'effectue selon les formules suivantes :

À partir de l’équation du bilan thermique :

La difficulté de déterminer la température de combustion à l'aide de cette formule est que la capacité calorifique du gaz dépend de la température. Puisque les gaz sont chauffés par la température T 0 jusqu'à la température Tg, alors dans la formule (36), il est nécessaire de substituer la valeur moyenne de la capacité thermique exactement dans cette plage de température. Mais la température de combustion nous est inconnue et nous souhaitons la connaître. Dans ce cas, vous pouvez procéder comme suit. La température moyenne de combustion de la plupart des substances dans l'air est d'environ 1 500 K. Par conséquent, avec une petite erreur de détermination Tg pour les calculs, vous pouvez prendre la valeur moyenne de la capacité thermique dans la plage de température de 273 à 1 500 K. Ces valeurs pour les principaux produits de combustion sont données dans le tableau. 6.

Tableau 6

Valeurs moyennes des capacités calorifiques des principaux produits de combustion dans la plage de température 273-1500 o C





La capacité thermique moyenne de certaines substances gazeuses dans différentes plages de température est également indiquée dans le tableau. III applications.

Regardons des exemples de résolution de problèmes de calcul de la température de combustion.

TÂCHES POUR UNE SOLUTION INDÉPENDANTE

1. Dans quel cas, en cas d'incendie, lors de la combustion du butane, plus de chaleur sera-t-elle libérée : en cas de combustion complète ou de combustion incomplète, en procédant par réaction C 4 H 10 + 4,5 O 2 4СО+5Н 2О. La réponse doit être confirmée par un calcul utilisant la loi de Hess.

2. Calculez la chaleur de formation de l'acétylène à partir des éléments si sa chaleur de combustion est de 1 411,2 kJ/mol.

3. Déterminez la chaleur de combustion de 12 kg de benzène, si la chaleur de sa formation est de 82,9 kJ/mol, la chaleur de formation de dioxyde de carbone est de 396,9 kJ/mol, la chaleur de formation de vapeur d'eau est de 242,2 kJ/mol .

4. Déterminez la chaleur de formation de l'acide pimélique (C 7 H 12 O 4), si sa chaleur de combustion est de 3453,5 kJ/mol.

5. Déterminez la chaleur de combustion de l'acide salicylique si la chaleur de sa formation est de 589,5 kJ/mol.

6. Calculez la chaleur de formation du méthane si 556,462 kJ de chaleur sont libérés lors de la combustion de 10 g de celui-ci dans des conditions standard.

7. Déterminez la chaleur de combustion de l'alcool benzylique (C 7 H 8 O), si la chaleur de sa formation est de 875,4 kJ/mol.

8. Lorsque l'octane se forme, 208,45 kJ/mol de chaleur sont libérés par les éléments. Calculez sa chaleur de combustion.

9. La chaleur de formation de l’acétone est de -248,28 kJ/mol. Déterminez sa chaleur de combustion et la quantité de chaleur qui sera dégagée lors de la combustion de 30 g de la substance.

11. Déterminer la chaleur de combustion de la sulfophénylhydrazine (C 6 H 8 O 3 N 2 S) en tenant compte des pertes dues à l'évaporation de l'eau. La teneur en humidité de la substance est de 20 %.

12. Déterminer la chaleur de combustion de la 4, 4/-diaminodiphénylsulfone (C 12 H 12 O 2 N 2 S) sans tenir compte des pertes dues à l'évaporation de l'humidité.

13. Déterminez la chaleur de combustion du 4, 6-diméthylhexahydro-1, 3, 5-triazinethione-2 (C 5 H 9 N 3 S) en utilisant les formules de D. I. Mendeleev.

14. Déterminez la chaleur de combustion de l'acide diaminomésitylène-6-sulfonique (C 9 H 14 O 3 N 2 S) si la teneur en humidité de la substance est de 35 %.

15. Déterminer la chaleur inférieure de combustion de la composition du bois : C – 41,5 % ; N – 6 % ; O – 43 % ; N – 2 % ; W– 7,5%.

16. Déterminez la température théorique de combustion de l'acétone en utilisant les capacités thermiques moyennes.

17. Déterminez la température théorique de combustion du pentane en utilisant les capacités thermiques moyennes.

18. Déterminez la température théorique de combustion de l'octane en utilisant les capacités thermiques moyennes.

19. Déterminez la température théorique de combustion du benzène en utilisant les capacités thermiques moyennes.

20. À l'aide de la méthode des approximations successives, calculez la température de combustion adiabatique du propanol.

21. Calculez la température de combustion pour un mélange stoechiométrique d'une substance combustible avec de l'air (tableau 7).

Tableau 7

22. À l'aide de la méthode des approximations successives, calculez la température de combustion réelle d'une substance combustible (tableau 8), si la combustion se produit avec un coefficient d'excès d'air α et que la fraction de perte de chaleur par rayonnement est η.

Tableau 8

Numéro de travail Nom de la substance Composition élémentaire de la substance, masse. % α η
C H Ô S N W cendre
22.1 Anthracite 0,5 1,0 21,5 1,1 0,2
22.2 Schistes bitumineux 24,2 1,8 4,5 3,0 2,0 39,5 1,2 0,3
22.3 Kérosène 13,7 0,3 - - - 1,3 0,4
22.4 Essence 8,0 5,0 - 2,0 1,4 0,3
22.5 Huile de sel 86,0 12,0 1,2 0,8 - - - 1,5 0,2
22.6 Essence - - 1,6 0,3
22.7 Bois - - 1,7 0,4
22.8 Charbon - 1,8 0,3
22.9 Cérésine - - - - 1,7 0,2
22.10 Schistes bitumineux 1,6 0,3

23. Déterminer la température théorique de combustion de la composition de caoutchouc : AVEC = 80 %, N= 15%, S = 2%, O= 1%, N = 2 %.

24 . Déterminer la température de combustion réelle du papier avec la composition : C = 55 %, H = 25 %, N = 3 %, O = 15 %, H 2 O = 2 %, si la perte de chaleur due à la sous-combustion était η X=0,15, dû au rayonnement η izl=0,20.

25. Déterminer la température de combustion réelle de la composition plastique : C = 70 %, H = 20 %, N = 5 %, O = 2 %, les composants ininflammables (charges) s'élèvent à 3 %/, si la perte de chaleur due à la sous-combustion est importante à η X=0,20, dû au rayonnement η izl=0,25. Coefficient d'excès d'air α = 1,4.

APPLICATION

Liste des notations acceptées

n– nombre de moles de substance ;

β – coefficient stœchiométrique ;

V en théorie– théoriquement nécessaire à la combustion, m 3 ;

V en d– volume réel (pratique) d'air utilisé pour la combustion, m3 ;

V page t– volume théorique des produits de combustion, m3 ;

R.– pression du gaz, Pa ;

P 0– pression (atmosphérique) initiale, Pa ;

T– température de la substance, K ;

Q– quantité de chaleur, J ;

VI– volume de la ième substance gazeuse, m 3, kmol ;

α - coefficient d'excès d'air ;

m– masse de la substance, kg ;

M– masse d'un kmole de la substance, kg/kmol ;

Q n– chaleur inférieure de combustion de la substance, kJ/mol, kJ/kg ;

N je– enthalpie de la ième substance, kJ/mol, kJ/m 3 ;

Tg– température de combustion, K ;

срi– capacité calorifique du ième gaz à pression constante, kJ/mol*K ; kJ/m3 ;

η – coefficient de perte de chaleur.

Tableau I

Constantes physiques de base de certains gaz

Transcription

1 MINISTÈRE DE LA FÉDÉRATION DE RUSSIE POUR LA DÉFENSE CIVILE, LES SITUATIONS D'URGENCE ET L'ÉLIMINATION DES CATASTROPHES Académie des pompiers de l'État A.S. Androsov, E.P. Saleev EXEMPLES ET PROBLÈMES pour le cours THÉORIE DE LA COMBUSTION ET DE L'EXPLOSION Moscou 5

2 MINISTÈRE DE LA FÉDÉRATION DE RUSSIE POUR LA DÉFENSE CIVILE, LES SITUATIONS D'URGENCE ET L'ÉLIMINATION DES CATASTROPHES Académie des pompiers de l'État A.S. Androsov, E.P. Saleev EXEMPLES ET TÂCHES pour le cours THÉORIE DE LA COMBUSTION ET DE L'EXPLOSION Manuel approuvé par le ministère de la Fédération de Russie pour la défense civile, les urgences et les secours en cas de catastrophe comme outil pédagogique pour les établissements d'enseignement supérieur EMERCOM de Russie Moscou 5

3 UDC BBK A ISB N Androsov A.S., Saleev E.P. Exemples et tâches pour le cours. Théorie de la combustion et de l'explosion. Didacticiel. - M. : Service d'incendie de l'État Arfltvbz EMERCOM de Russie, p. Examinateurs : Département de chimie générale et spéciale de l'Académie des services d'incendie d'État du ministère des Situations d'urgence de Russie, Département des équipements et services d'incendie de l'Académie des services d'incendie d'État du ministère des Situations d'urgence de Russie. Les exemples et les tâches du cours Théorie de la combustion et de l'explosion sont compilés sur la base de nombreuses années d'expérience dans l'enseignement de la discipline à l'Académie des pompiers d'État du ministère des Situations d'urgence de Russie de manière à pouvoir servir de un guide pour la conception des cours. Afin d'assurer l'unité méthodologique avec la partie théorique du cours, des exemples de résolution de problèmes, ainsi que des formules de calcul de base, sont donnés au début de chaque chapitre. L'annexe contient les tableaux de valeurs les plus souvent utilisés lors de la résolution de problèmes dans ce cours. Destiné aux cadets, aux étudiants et aux auxiliaires des établissements d'enseignement du ministère russe des Situations d'urgence du profil technique d'incendie. Les chapitres 1 et 3 ont été rédigés par Ph.D. technologie. Professeur agrégé de sciences Androsov A.S., chapitre 4 Ph.D. technologie. Sciences, chercheur principal Saleev E.P. ISB N Académie des services d'incendie d'État du ministère des Situations d'urgence de Russie, 5

4 Contenu Chapitre 1. Bilans matériaux et thermiques des processus de combustion Calcul de la quantité d'air nécessaire à la combustion des substances Calcul du volume et de la composition des produits de combustion Calcul de la chaleur de combustion des substances Calcul de la température de combustion et d'explosion Chapitre. Limites de concentration de propagation de la flamme (inflammation) Chapitre 3. Indicateurs de température du risque d'incendie Calcul des limites de température de propagation de la flamme (inflammation) Calcul des températures d'éclair et d'inflammation Calcul de la température standard d'auto-inflammation. 61 Chapitre 4. Paramètres de l'explosion des mélanges vapeur-gaz Calcul de la pression maximale d'explosion Calcul de l'équivalent TNT de l'explosion et de la distance de sécurité par rapport à l'action des ondes de choc aériennes.. 63 Annexe

5 Chapitre 1. Bilans matières et thermiques des processus de combustion La base théorique du calcul des bilans matières et thermiques sont les lois fondamentales de la conservation de la matière et de l'énergie Calcul de la quantité d'air nécessaire à la combustion des substances Formules de calcul Pour les calculs pratiques, il est supposé cet air est composé de 1 % d’oxygène et de 79 % d’azote. Ainsi, le rapport volumétrique de l'azote et de l'oxygène dans l'air sera : φ φ Ο 79 3,76, (1.1) 1 Ν où φ Ν, φ Ο sont respectivement la teneur volumétrique (% vol.) d'azote et d'oxygène dans le milieu oxydant. . Ainsi, pour 1 m 3 (kmol) d'oxygène dans l'air, il y a 3,76 m 3 (kmol) d'azote. Le rapport massique de l'azote et de l'oxygène dans l'air est de 3,3 % d'O et de 76,7 % de N. Il peut être déterminé sur la base de l'expression : φ φ N O M M N O ,9, (1.) 1 3 où M, M O N sont les poids moléculaires de l'oxygène. et de l'azote, respectivement. Pour faciliter les calculs, les substances inflammables sont divisées en trois types (tableau 1.1) : composés chimiques individuels (méthane, acide acétique, etc.), substances de composition complexe (bois, tourbe, schiste, pétrole, etc.), mélange de gaz (gaz producteur, etc.). 5

6 Tableau 1.1 Type de substance inflammable Formules de calcul Dimension Substance individuelle B (1.3, a) n no + n kmol m 3 ; N kmol m 3 Substance de composition complexe Mélange de gaz G (no + nn) V ngm g (1,3, b) C S O V.69 + H (1,4) φg O i φo V in (1,5) 1 m 3 kg m 3 kg m 3 kmol; m 3 kmol Ici B est la quantité théorique d'air ; n Г, non, nn quantité de carburant, d'oxygène et d'azote obtenue à partir de l'équation de la réaction chimique de combustion, kmol ; M G poids moléculaire du carburant ; volume de 1 kmol de gaz dans des conditions normales (0,4 m3) ; Teneur massique en C, H, S, O des éléments correspondants dans la composition du carburant, % ; ϕ Г i concentration du i-ième composant combustible, % vol.; ϕ O concentration d'oxygène dans la composition du gaz combustible, % vol.; non i est la quantité d'oxygène nécessaire pour oxyder un kmole du i-ième composant combustible, kmol. Pour déterminer le volume d'air lors de la combustion dans des conditions autres que normales, utilisez l'équation d'état des gaz parfaits P T P T 1 1, (1.6) où P est la pression normale, Pa ; T température normale, K ; volume d'air dans des conditions normales ; P 1, 1, T 1 - respectivement pression, volume et température de l'air caractérisant les conditions de combustion données. Quantité d'air pratique Volume d'air entrant réellement dans la zone de combustion. Le rapport entre le volume d'air pratique et le volume théorique est appelé coefficient d'excès d'air α : 1 V α. (1.7) La différence entre les volumes d'air pratique et théorique est appelée excès d'air B : B B B B. (1.8) 6

7 D'après les équations (1.7) et (1.8), il s'ensuit que B B (α-1). (1.9) Si la teneur en oxygène dans les produits de combustion est connue, alors le coefficient d'excès d'air est déterminé par la formule φ α 1+ O B (1 φ) O, (1.1) où ϕ O est la concentration d'oxygène dans les produits de combustion, % vol. ; volume théorique de produits de combustion. Pour les substances dans lesquelles le volume de produits de combustion est égal au volume d'air consommé (par exemple, le carbone), la formule (1.1) est simplifiée : 1 α. (1.11) 1 Dans le cas de formation de produits de combustion incomplète (CO, H, CH 4, etc.), la formule (1.11) prend la forme φ O 1 α 1 φ, (1.11, a) O +,5φ ,5φ φ CO + H + CH 4 où φ O, φ co, φ CH, φ 4 H la teneur des substances correspondantes dans les produits de combustion, % vol. Si la teneur en oxygène dans un environnement oxydant diffère de sa teneur dans l'air, alors la formule (1.1) peut s'écrire sous la forme : α 1+ et, par conséquent, la formule (1.11) φ O (φ φ) O O O O (1.1) φo α, ( 1.13) φ φ où φ O est la teneur initiale en oxygène dans le milieu oxydant, % vol. ; volume théorique du milieu oxydant. Souvent, dans les calculs techniques d'incendie, il est nécessaire de déterminer la masse d'air utilisée pour la combustion, où ρ en densité de l'air, kg/m 3. Évidemment, m in en ρ in, (1.14) 7

8 ρ φ М + φ М PT N N O O B. (1.15) PT 1 Après avoir substitué des valeurs constantes dans la formule (1.15), nous obtenons 3 P ρв 3, 47 1, (1.16) T où P est la pression atmosphérique, Pa ; T température de l'air, K. Exemples Exemple 1. Déterminer la masse et le volume théoriques d'air nécessaires à la combustion de 1 m 3 de méthane dans des conditions normales. Solution Une substance inflammable est un composé chimique individuel, par conséquent, pour calculer le volume d'air, vous devez utiliser la formule (1.3, a). Écrivons l'équation de la réaction chimique de combustion de CH 4 dans l'air CH 4 + O + 3,76 N CO + HO + 3,76 N. À partir de l'équation, nous trouvons n O ; n 3,76 7,5 ; N n 1 CH, puis 4 + 7,5 B 9,5 m 3 /m 3 ou kmol/kmol. 1 À l'aide de la formule (1.14), en tenant compte de l'équation (1.15), on calcule la masse d'air, 79 8 +.1 3 m B 9,5 9,5 1,8 1, kg/m 3.,4 Exemple. Déterminer le volume théorique d'air nécessaire à la combustion de 1 kg de benzène. Solution Un composé chimique individuel inflammable, donc pour calculer à l'aide de la formule (1.3, b), nous écrivons l'équation de la réaction chimique de combustion C 6 H 6 + 7,5 O + 7,5 3,76 N 6 CO + 3 HO + 7,5 3,76 N , trouver n 1 ; n 7 5 ; n 7,5 3,76 8,. O, N Masse moléculaire du benzène M Le volume de 1 kmol de gaz dans des conditions normales est de 4 m 3 (7,5 +,) 8,4 B 1,3 m 3 /kg

9 EXEMPLE 3. Déterminer le volume et la masse d'air nécessaires à la combustion de 1 kg de masse organique de la composition : C 6%, H 5%, O 5%, N 5%, W 5% (humidité), si l'excès coefficient air α,5; température de l'air 35 K, pression 995 Pa. Solution Étant donné que la substance inflammable a une composition complexe, la quantité théorique d'air dans des conditions normales est déterminée par la formule (1.4) 6 5 V, 9 m 3 /kg. 3 8 À partir de la formule (1.7), nous calculons la quantité pratique d'air dans des conditions normales α, 5 5, m 3 /kg. En B, on retrouve la quantité d'air utilisée pour la combustion de la substance dans des conditions de combustion données. En utilisant la formule (1.6), nous obtenons m 14, V (RT) 16,8 m 3 /kg, 8 1, (RT) V ρ V 18,9 V kg/kg. EXEMPLE 4. Déterminer le volume d'air nécessaire à la combustion de 5 m 3 d'un mélange de gaz constitué de % CH 4 ; 4% CH; 1% de CO ; 5% N et 5% O si le taux d'excès d'air est de 1,8. Solution Le carburant est un mélange de gaz, donc pour calculer le volume d’air utilisé pour la combustion, nous utilisons la formule (1.5). Pour déterminer les coefficients stoechiométriques pour l'oxygène no i, nous écrivons l'équation des réactions de combustion des composants inflammables dans l'oxygène CH 4 + O CO + H O, CH +.5O CO + H O, CO +.5O CO, +, 5 4 + , puis B 5,7 m 3 /m 3. 1 Pour la combustion de 5 m 3 de mélange gazeux, le volume d'air théorique requis sera de B 5 5, 7 8, 5 m 3. Quantité d'air pratique : 18, 3, m 3.B.9

10 EXEMPLE 5. Déterminer le coefficient d'excès d'air lors de la combustion de l'acide acétique, si 3 m 3 d'air étaient fournis pour la combustion de 1 kg. Solution Pour déterminer le coefficient d'excès d'air à l'aide de la formule (1.7), il est nécessaire de calculer sa quantité théorique. Le poids moléculaire de l'acide acétique est de 6. CH 3 COOH + O + 3,76 N CO + HO + 3,76 N ; (+,) 3 76,4 B 3,6 m 3 /kg. 1 6 Alors le coefficient d'excès d'air selon la formule (1.7) est égal à 3, α, 8. 3, 6 La combustion s'est produite avec un manque d'air. EXEMPLE 6. Déterminer le volume d'air utilisé pour oxyder 1 m 3 d'ammoniac si la teneur en oxygène dans les produits de combustion était de 18 %. Solution On détermine la quantité théorique d'air nécessaire à la combustion de 1 m 3 d'ammoniac : alors NH 3 +.75O +.75 3.76N,5N + 1.5H O +.75 3.76N, 75+, 75 3, 76 B 3, 6 m 3 / m 3. 1 Pour déterminer le coefficient d'excès d'air à l'aide de la formule (1.1), il est nécessaire de calculer la quantité théorique de produits de combustion de 1 m 3 d'ammoniac (1., formule 1.14) 1,5 + 0,5 +, 75 3,76 4,8 m 3 /m 3. 1 Coefficient d'excès d'air 18 4, 8 α 1+ 9,. 3, 6 1 (18) Le volume d'air impliqué dans le processus de combustion de 1 m 3 d'ammoniac est déterminé à partir de la formule (1.7) 9 3, m 3 / m 3. V, 1

11 EXEMPLE 7. Déterminer le volume du milieu oxydant, constitué de 6 % d'O et 4 % de N, nécessaire à la combustion de 1 kg d'alcool isopropylique, si sa température est de 95 K, pression 6, kPa. Solution Étant donné que la composition du milieu oxydant diffère de celle de l’air, nous déterminons à l’aide de la formule (1.1) le rapport volumique de l’oxygène et de l’azote est de 4 : 6,67. L'équation de réaction de combustion pour l'alcool isopropylique est C 3 H 7 OH + 4,5O + 4,5,67N 3CO + 4HO + 4,5,67N. Le volume théorique du milieu oxydant dans des conditions normales sera calculé à l'aide de la formule (1.3, b). Le poids moléculaire du carburant est de 6 : (4,5 + 4,5,67) 1 6,4 os.8 m 3 /kg. Le volume du milieu comburant dans des conditions de combustion données sera déterminé à partir de la formule (1.6) () 4, 9 os RT, 35 6, 73 m 3 /kg. Exemple 8. Déterminer la masse de dinitrotoluène, C 7 H 6 (NO), brûlé dans un volume scellé de 1 m 3, si la teneur en oxygène dans les produits de combustion était de 1 %. Solution Étant donné que les produits de combustion contiennent de l’oxygène, la combustion s’est produite dans un excès d’air. Nous déterminons le coefficient d'excédent à l'aide de la formule (1.1). C 7 H 6 (NON) + 6,5O + 6,5 3,76N 7CO + 3HO + N + 6,5 3,76N. Masse moléculaire du carburant 18. Volume théorique d'air (6, 5 + 6, 5 3, 76), 4 V 38, m 3 / kg Volume théorique des produits de combustion (formule 1.14) (, 5,) 3 76, 4 4 , 4 m 3 /kg, 4 α 1+, 1 (1) Le volume pratique d'air utilisé pour la combustion est de 55 38,9 7 m 3 /kg. À 11 HEURES

12 Ensuite, nous déterminons la masse de dinitrotoluène brûlé m g à partir du rapport P 1 m G 1,3 kg. 9,7 V Tâches de test 1. Déterminer la masse et le volume (théorique) d'air nécessaire à la combustion de 1 kg d'alcools méthylique, éthylique, propylique et amylique. Construire un graphique du volume d'air en fonction du poids moléculaire de l'alcool.Déterminer le volume d'air théorique nécessaire à la combustion de 1 m 3 de méthane, éthane, propane, butane et pentane. Construire un graphique de la dépendance du volume d'air sur la position de la substance dans la série homologue (teneur en carbone dans la molécule de la substance). 3. Déterminez la masse théorique d'air utilisée pour brûler 1 kg de méthane, d'alcool méthylique, d'aldéhyde formique, d'acide formique. Expliquer la raison de l'influence de la composition de la substance sur le volume d'air nécessaire à leur combustion. 4. Déterminer le volume et la masse d'air utilisé pour la combustion de 1 kg de bois de composition : C 47%, H 8%, O 4%, W 5%, si le coefficient d'excès d'air est de 8 ; pression 9 GPa, température 85 K. 5. Quelle quantité d'air, en kg, a été fournie pour la combustion de 1 kg de carbone, si la teneur en oxygène dans les produits de combustion était de 17 % ? 6. Quelle quantité d'air, en kg, doit être fournie pour la combustion de m 3 de gaz générateur de composition : CO 9%, H 14%, CH 4 3%, CO - 6,5%, N - 45%, O - .5%, si le coefficient d'excès d'air est égal à 5 ​​? 7. Déterminez la quantité de toluène brûlé, en kg, dans une pièce d'un volume de 4 m 3 si, après un incendie en l'absence d'échange gazeux, il est déterminé que la teneur en oxygène a diminué à 17 %. 8. Quelle quantité de chlore, en m3, a été fournie pour la combustion de 3 m3 d'hydrogène, si l'excès de comburant dans les produits de combustion était de 8 m3 ? 9. Déterminer l'excès d'air dans les produits de combustion d'un mélange gazeux de composition : CO 15 %, C 4 H 1 45 % O 3 %, N 1 %, si le coefficient d'excès d'air est de 1,9. 1. Quelle quantité de milieu oxydant, m3, composé de 5 % d'oxygène et 5 % d'azote, est nécessaire pour la combustion de 8 kg d'acétate d'éthyle, si le coefficient d'excès est de 1 ; température 65 K, pression 85 GPa. 11. Déterminer le coefficient d'excès du milieu comburant, constitué de 7% d'oxygène et 3% d'azote, si lors de la combustion du soufre la teneur est de 1

13 l'oxygène a diminué à 55%. Déterminez la quantité de soufre brûlé (kg) si le volume de la pièce est de 18 m3. Quelle quantité d'anthracite (en supposant que la teneur en carbone est de 1%) a brûlé dans une pièce d'un volume de 15 m3, si la combustion s'est arrêtée lorsque l'oxygène a diminué à 13%. Ignorer les échanges gazeux. 13. Calculer le débit d'air massique et volumétrique nécessaire à la combustion d'une fontaine à gaz avec un débit de 3 millions de m 3 /jour, composé de CH 4,8 %, CO 1 %, H S 5 %, O 5 % à une température de l'air de 7 o C et une pression de 15 kpa. Devoir : Calculer le volume et la masse du milieu comburant nécessaire à la combustion de la ième substance inflammable (tableau 1). Numéro d'option Substance combustible Formule chimique Quantité de combustible Composition du milieu comburant 1 Alcool méthylique CH 3 OH kg Air Aniline 3 Mélange de gaz 4 Nitrobenzène 5 Substance complexe C 6 H 7 N CO 45% N 15% C 4 H 8 1% O 3% 5 kg O 7% N 3% Tableau 1. Conditions de combustion T 3 K P1135 Pa α 3 T 9 K P 9 Pa α,5 3 m 3 Air Normal α 1,8 C 6 H 5 NO 3 kg Air C 65 % O % H 5% S 1% T 8 K P 98 Pa α,5 g Air Normal α 1,4 6 Éthylène C H 4 5 m 3 O 5% N 75% 7 Soufre O 6% S kg N 4% 8 Substance complexe C 9% H 3 % N 5% O% 1 kg Air Normal α,5 T 35 K P1 Pa α 1,8 T 3 K P 95 Pa α 1,5 13

14 Numéro d'option 9 Mélange gazeux Substance combustible Formule chimique CH 4 15% C 3 H 8 7% O 1% H 5% Suite du tableau 1. Quantité Composition comburante Conditions de combustion combustible 5 m 3 Air Normal α 1,9 1 Aluminium Al 15 kg O 4 % N 58% Normal α.8 11 Alliage Mg% Al 8% 8 kg Air T 65 K P 9 Pa α 1.5 1 Acide formique C O 5 kg Air Normal α 1, 13 Éther diméthylique (CH 3) O 1 kg Air 14 Mélange de gaz 15 Substance complexe 16 Glycérol HS 5% SO 15% CO 15% H 3% O 15% C 8% H 8 % W 1% T 8 K P116 Pa α 4, 15 m 3 Air Normal α 1,4,7 kg Air C 3 H 8 O 3 1 kg Air 17 Acétylène C H 15 l Cl 18% N 8% 18 Mélange de gaz 19 Ester d'acide éthylacétique Méthyléthylcétone 1 Chlorobenzène Nitrotoluène CH 4 3% O 8% N 15% H 47% T 6 K P11 Pa α 1,4 T 35 K P113 Pa α 1,9 Normal α 1,8 3 m 3 Air Normal α 3, C 4 H 8 O 5 kg Air T 7 K P 85 Pa α 1,5 C 4 H 8 O 5 kg Air Normal α .5 T 35 K C 6 H 5 Cl 7 kg Air P 1 Pa α.8 C 7 H 7 NO 1 kg O 5% N 75% T 8 K P 98 Pa α 1.4 14

15 Option numéro 3 Mélange gazeux Substance combustible Formule chimique NH 3 5% C 4 H 1 5% C 4 H 8 15% CO 3% O 5% Quantité de substance inflammable Fin du tableau 1. Composition du milieu de combustion comburant Air 4 Butyle alcool C 4 H 1 O 4 kg Air 5 Dibromohexane C 6 H 1 Br 3 kg 6 Substance complexe 7 Mélange de gaz C 7% S 5% H 5% O% C 3 H 8 1% CO 79% H 5% O 5 % N 1% O 65% N 35% 15 kg Air Normal α 1,8 T 65 K P1 Pa α 1,8 T 8 K P 98 Pa α 1,7 T 85 K R 1 Pa α.8 1 m 3 Air Normal α 3,5 1.. Calcul de le volume et la composition des produits de combustion Afin de simplifier le calcul, toutes les substances inflammables sont divisées en trois types : individuels, complexes, mélanges de gaz combustibles (tableau 1.3) Type de substance inflammable Substance individuelle Substance de composition complexe n Formules de calcul (1.17 ) ng ni (1,18) n M G C CO 1,86 1 (1,19) H W H O 11, + 1,4 1 1 (1.) S SO,7 1 ( 1,1) 1 7C+ 1 H O +.63S +.8 N (1.) N 1 8 Tableau 1.3 Dimension m 3 ; kmol m 3 kmol m 3 kg m 3 ; kmol kg kg 15

16 Type de substance inflammable Mélange de gaz i Formules de calcul i 1 ni φн ii + φнgi. 1 ng (1,3) Dimension m 3 ; kmol m 3 kmol Voici le volume théorique des produits de combustion ; n НГi est la quantité du i-ème produit de combustion dans l'équation de réaction, kmol ; n Г quantité de carburant, kmol ; volume de 1 kmol de gaz ; M poids moléculaire du carburant ; НГi est le volume du i-ème produit de réaction ; Teneur en C, H, S, O, N, W des éléments correspondants (carbone, hydrogène, soufre, oxygène et azote) et humidité dans la substance combustible, % en poids ; ϕ Гi teneur du i-ième composant combustible dans le mélange gazeux, % vol. ; ϕ НГi teneur du ième composant ininflammable dans le mélange gazeux, % vol. Le volume pratique (total) des produits de combustion est constitué du volume théorique des produits de combustion et de l'excès d'air ou + (α 1) + Δ (1,4) V V. (1,5) Composition des produits de combustion, c'est-à-dire la teneur en ième composant est déterminée par la formule φ i 1, (1.6) i i où ϕ i est la teneur en ième composant dans les produits de combustion, % vol. ; je volume du ième composant, m 3, kmol ; Σ je volume total de produits de combustion, m 3, kmol. Lors de la combustion dans un excès d'air, les produits de combustion contiennent de l'oxygène et de l'azote O.1ΔB ; (1.7) N + 79, (1.8) N, B où N est le volume théorique d'azote dans les produits de combustion, m 3, kmol, 79. (1.9) N B 16

17 Exemples Exemple 1. Quelle quantité de produits de combustion sera libérée lors de la combustion de 1 m 3 d'acétylène dans l'air si la température de combustion est de 145 K. Solution Un composé chimique individuel inflammable (formule 1.17). Écrivons l'équation de la réaction chimique de combustion : CH +,5O +,5 3,76N CO + H O +,5 3,76N Volume des produits de combustion dans des conditions normales + 1+,5 3,76 1,4 m/m 3. 1 Volume de produits de combustion à 145 K 1,4 145 (RT) 65,9 m 3 / m EXEMPLE. Déterminez le volume de produits de combustion lors de la combustion de 1 kg de phénol, si la température de combustion est de 1 K, la pression est de 95 Pa et le coefficient d'excès d'air est de 1,5. Solution Composé chimique individuel inflammable (formule 1.18). Écrivons l'équation de la réaction chimique de combustion C 6 H 5 OH + 7O + 7 3,76N 6CO + 3H O + 7 3,76N. Masse moléculaire du carburant 98. Volume théorique des produits de combustion dans des conditions normales () 3,76,4 8,1 m 3 /kg Volume pratique d'air dans des conditions normales (1,5) (.76)(1,5 1) 8,1 +, 4 11,9 m 3 / kg Volume de produits de combustion dans des conditions données 11, (RT) 55,9 m 3 /kg Exemple 3. Déterminer le volume de produits de combustion lors de la combustion de 1 kg de composition de masse organique : C 55%, O 13%, H 5%, S 7%, N 3%, W 17%, si la température de combustion est de 117 K, le coefficient d'excès d'air est de 1,3. 17

Solution 18. Une substance inflammable de composition complexe (formule). La composition théorique des produits de combustion dans des conditions normales est de 55 CO 1,86 1, m 3 /kg ; H O 11, + 1,4,6 +,8 m 3 /kg; SO,7,5 m 3 /kg ; N 0,63+0,8 3 4,7 m 3 /kg. 1 8 Le volume théorique total des produits de combustion dans des conditions normales est de 1+,8 +,5 + 4,7 6,55 m 3 /kg. Le volume pratique des produits de combustion dans des conditions normales est de 55+, (1,3 1) 6,55+ 1,8 8, 35 m 3 /kg. 3 8 Le volume pratique des produits de combustion à une température de combustion de 8. (RT) 35,8 m 3 /kg. 73 Exemple 4. Calculer le volume de produits de combustion lors de la combustion de 1 m 3 d'un mélange gazeux constitué de C 3 H 6 7%, C 3 H 8 1%, CO 5%, 15%, si la température de combustion est de 13 K, coefficient d'excès d'air, 8. Température ambiante 98 K. Solution. Carburant - mélange de gaz (formule 1.3) C 3 H 6 + 4,5O + 4,5 3,76N 3CO + 3H O + 4,5 3,76N, C 3 H 8 + 5O + 5 3,76N 3CO + 4HO + 5 3,76N. Le volume des produits de combustion est déterminé par la formule (1.3) 1 CO (), 45 m 3 / m 3 ; 1 1 H O (), 4 m 3 / m 3. 1 Étant donné que le mélange gazeux contient de l'oxygène, il oxydera certains des composants inflammables, par conséquent, le débit d'air diminuera (formule 1.5). 18

19 Dans ce cas, il est plus pratique de déterminer le volume théorique d'azote à l'aide de la formule (1.9) 4, N, 79 13, m 3 / m 3. 1 Volume théorique des produits de combustion, 45 +, 4 + 13, 18,5 m 3 / m 3. Volume pratique des produits de combustion (formules 1.4, 1.5) 4,5 + (.8 1) 4,5 m 3 / m 3. 1 Volume des produits de combustion à une température de 13 K 4,5 13 (RT) 183,4 m 3 / m P EXEMPLE 5. Déterminer la composition des produits de combustion de la méthyléthylcétone. Solution Avec cette formulation du problème, il est plus rationnel de déterminer directement à partir de l'équation de combustion le volume de produits en kmoles libérés lors de la combustion de 1 kmole de carburant CH 3 COC H 5 + 5,5O + 5,5 3,76N 4CO + 4HO + 5,5 3,76N, CO 4 kmol ; 4 H O kmole ; N, 7 kmol; 7 je 8, kmol. A l'aide de la formule (1.6), on trouve la composition des produits de combustion 4 1 ϕ H 14 O ϕco %, ϕ N 1 () 7 %. 8.7 Exemple 6. Déterminer le volume et la composition (% vol.) des produits de combustion de 1 kg d'huile minérale de composition : C 85%, H 15%, si la température de combustion est de 145 K, le coefficient d'excès d'air est de 1,9. Solution À l'aide des formules (), nous déterminons le volume des produits de combustion 85 CO 1,86 1,6 m 3 /kg ; 1 15 HO 11, 1,7 m 3 /kg; 1 1 N () 9,1 m 3 /kg. 1 Volume théorique des produits de combustion dans des conditions normales 19

20 1,6 + 1,7 + 9,1 1,4 m 3 /kg. Le volume pratique des produits de combustion dans des conditions normales (formule 1.5) est de 85 1,4 +, (1,9 1) 1,4 + 1,5, 9 m 3 /kg. 3 Le volume des produits de combustion à une température de 145 K.9 145 (RT) 11,7 m 3 /kg. 73 Évidemment, la composition des produits de combustion ne dépend pas de la température de combustion, il convient donc de la déterminer dans des conditions normales. D'après les formules (1.6, 1.8) 1,6 1,1 1,5 1 ϕ CO 7,1 % ; ϕ 9, 4 O % ; .9,9 (9,1 + .79 1,5) 1 1,7 1 ϕ N 76, % ; ϕ 7, 3 H O,9%.,9 Exemple 7. Déterminez la quantité d'acétone brûlée, en kg, si le volume de dioxyde de carbone libéré, ramené aux conditions normales, était de 5 m 3. Solution. Écrivons l'équation de réaction de combustion de l'acétone dans l'air CH 3 COCH 3 + 4O + 4 3,76N 3CO + 3HO + 4 3,76N. De l'équation, il résulte que lors de la combustion, 3,4 m 3 de dioxyde de carbone sont libérés à partir de 58 kg (poids moléculaire de l'acétone). Ensuite, pour former 5 m 3 de dioxyde de carbone, m G de carburant doit réagir 5,58 m G 43, kg. 3.4 Exemple 8. Déterminer la quantité de masse organique brûlée de la composition : C 58%, O%, H 8%, N%, W 1% dans une pièce d'un volume de 35 m 3, si la teneur en dioxyde de carbone était de 5 %. Solution Déterminons le volume de dioxyde de carbone libéré Ex 35,5 17,5 m 3. CO En utilisant la formule (1.19) pour une substance de composition complexe, nous déterminons le volume de CO libéré lors de la combustion de 1 kg de carburant, 58 CO 1,86 1 . 1 m 3 /kg. 1

21 Déterminons la quantité de substance brûlée : 17,5 m D 15,9 kg. 1.1 Exemple 9. Déterminer le moment où la teneur en dioxyde de carbone dans une pièce d'un volume de 48 m 3 suite à la combustion du bois (C 45 %, H 5 %, O 4 %, W 8 %) était de 8 %, si le taux massique spécifique de combustion du bois est de 8 kg/(m s) et la surface de combustion est de 38 m. Lors de la décision, ne tenez pas compte des échanges gazeux avec l'environnement et négligez la dilution résultant du dégagement de produits de combustion. Solution La dilution par les produits de combustion n'étant pas prise en compte, on détermine le volume de dioxyde de carbone libéré suite à la combustion, correspondant à 8% de sa teneur dans l'atmosphère 8 48 CO 38,4 m 3 1 à partir de l'expression (1.19) nous déterminer la quantité de matériau combustible qui doit être brûlée pour libérer un volume donné de dioxyde de carbone 38,4 m D 46 kg. 1.86.45 Nous déterminons la durée de combustion sur la base de la relation m τ, υ Г F m où τ est la durée de combustion ; m G masse de bois brûlé, kg ; υ m taux massique de combustion du bois, kg/(m s) ; F surface de combustion, m ; 46 τ 151 s.5 min.8 38 Tâches de test 1. Déterminer le volume et la composition (% vol.) des produits de combustion de 1 m 3 d'éthylène, propylène, butylène, si la température de combustion est de 18 K, la pression est 98 Pa. Dessiner un graphique de la dépendance du volume des produits de combustion et de la teneur en composants individuels sur le poids moléculaire du carburant. Déterminer le volume des produits de combustion et la teneur en vapeur d'eau et d'oxygène lors de la combustion de 1 kg d'hexane, heptane , octane, décane, si la température de combustion est de 13 K, pression GPa, coefficient d'excès 1

22 air comburant 1.8. Dessinez un graphique de la dépendance du volume des produits de combustion et de la teneur en oxygène sur le poids moléculaire du carburant. 3. Déterminer le volume et la composition des produits de combustion de 1 kg de bois de composition C 49%, H 6%, O 44%, N 1%, si la température de combustion est de 15 K, le coefficient d'excès d'air est de 1,6. 4. Combien de produits de combustion, ramenés aux conditions normales, se forment à la suite de la combustion de 5 m 3 d'un mélange gazeux de composition H 45%, C 4 H 1%, CO 5%, NH 3 15%, O 15%, si la combustion s'est produite avec un coefficient d'excès d'air égal à 3 ? 5. Déterminez la quantité de pétrole brut de composition : C 85 %, H 1 %, S 5 % brûlé dans un volume de 5 m3, si la teneur en dioxyde de soufre était de 5 m3. Calculez à quelle teneur en oxygène la combustion s'est arrêtée. 6. Au bout de quel temps la teneur en CO dans une pièce d'un volume de 3 m 3 suite à la combustion de l'hexanol sur une surface de 8 m sera-t-elle de 7 % ? Taux de combustion massique de l'hexane, 6 kg/(m s). 7. Déterminer la teneur en SO (% vol.) dans un volume de 1 m 3 pour,5 m et 4 minutes de combustion d'huile de composition : C 8%, H 8%, S 1%, si son taux de combustion de une superficie de 5 m était de 0,4 kg /(m s). Construisez un graphique de la dépendance de la teneur en dioxyde de soufre sur le temps de combustion. 8. Déterminer le volume de produits de combustion dégagés à la 5ème minute après allumage du mélange gazeux de la composition : C H 3%, H%, O 15%, HS 18%, CO 15% et la teneur en dioxyde de carbone, si le le coefficient d'excès d'air est de 1,5, température de combustion 13 K. Débit de gaz 5 m 3 /s, température du gaz 95 K. Devoirs Calculer le volume de produits formés, m 3, et leur teneur en azote (% vol.) pendant la combustion de la ième substance (tableau 1.4). Tableau 1.4 Numéro d'option Substance combustible Formule chimique Quantité de combustible Composition du milieu comburant 1 Alcool diéthylique (CH 5) O 1 kg Air Acide acétique C H 4 O 5 kg « 3 Alliage Mg% Al 8% 1 kg « Conditions de combustion T g 15 K P114 Pa α,5 T g 1 K P 98 Pa α,6 T g 8 K P 95 Pa α 1,6

23 SUITE TABLEAU. 1.4 Option numéro 4 Mélange de gaz Substance combustible Formule chimique CH 4% C 3 H 8 65% O 15% Quantité de combustible Composition du milieu comburant 1 m 3 "5 Alcool octylique C 8 H 18 O 1 kg "6 Substance complexe 7 Mélange de gaz 8 Aniline C 9% H 5% O 5% NH 3 1% C 4 H 1 8% N 7% O 3% 1 kg "1 m 3 "C 6 H 7 N 1 kg "9 Éther diéthylique (CH 5) O 5 kg "1 Mélange de gaz 11 Nitrobenzène 1 Substance complexe 13 Mélange de gaz CO 7% C 3 H 8 5% O 5% C 6 H 5 NO C 7% H 6% O 14% W 1% CH 4 6% CO 3% H 1% 14 Éther diméthylique (CH 3) O 1 kg 15 Glycérol 16 Substance complexe C 3 H 8 O 3 C 8% H 1% O 8% 1 m 3 O 4% N 58% kg Air 1 kg “1 m 3 “1 kg O 3% N 7% O 7% N 73% 1 kg Air Conditions de combustion T g 148 K P113 Pa α,4 T g 13 K P1 Pa α,5 T g 13 K P 97 Pa α 1,6 T g 16 K P113 Pa α 1, T g 155 K P 94 Pa α 1,7 T g 16 K P113 Pa α 1,4 T g 14 K P113 Pa α,5 T g 18 K P 87 Pa α 1,8 T g 13 K P 97 Pa α 1,3 T g 15 K P113 Pa α 1, T g 18 K P 87 Pa α 1,8 T g 16 K P113 Pa α,1 T g 135 K P 99 Pa α 1,8 3

24 SUITE TABLEAU. 1.4 Option numéro 17 Mélange de gaz Substance combustible Formule chimique C H 6 6% C 3 H 8 3% H 5% O 5% Quantité de combustible Composition du milieu comburant 1 m 3 - « - 18 Méthyléthylcétone C 4 H 8 O 1 kg - "- 19 Substance complexe Nitrotoluène 1 Mélange de gaz C 6% H 7% O - 1% W 1% 4 kg - "- C 7 H 7 NO kg - "- NH 3 4% C 3 H 8 4% H 1% O 1% Dibromohexane C 6 H 1 Br 1 kg 1 m 3 - "- O 5% N 5% 3 Dinitrobenzène C 6 H 4 (NO) 1 kg Air 4 Disulfure de carbone CS kg - "- 5 Dichlorobenzène C 6 H 4 Cl 5 kg - « - 6 Acide formique 7 Acétate d'éthyle C 7% S 5% H 5% O% 1 kg O 8% N% C 4 H 8 O 1 kg Air Conditions de combustion T g 165 K P113 Pa α. 6 T g 148 K P 91 Pa α 1,7 T g 11 K P113 Pa α 1,4 T g 134 K P1 Pa α,6 T g 18 K P113 Pa α 1,7 T g 14 K P 9 Pa α,3 T g 165 K P 81 Pa α 1,1 T 17 K P 97 Pa α 1,6 T 13 K P 99 Pa α 1,4 T 6 K P 98 Pa α,5 T g 15 K P1 Pa α 1,5 4

25 1.3. Calcul de la chaleur de combustion des substances Formules de calcul Lors du calcul du bilan thermique dans un incendie, la chaleur de combustion la plus basse est généralement déterminée. La quantité de chaleur dégagée lors de la combustion d'une unité de masse (volume) de combustible à l'état gazeux de l'eau Q Q Q, V H où Q est le pouvoir calorifique supérieur de la combustion ; Qn pouvoir calorifique inférieur ; Q est la chaleur d'évaporation de l'eau formée lors de la combustion d'une substance. is Type de combustible Substances Substances individuelles Substances de composition complexe (formule de Mendeleïev) Mélange de gaz Tableau 1.5 Formules de calcul Dimension Q (ni H n H) (1,3) kJ/mol H i ​​i j QН 339,4C + 157H 18, 9(O S) 5,1 (9H + W) (1,31) 1 Q H QНφгi (1,3) 1 kJ/kg kJ/mol ; kJ/m 3 où H i, H j, respectivement, la chaleur de formation d'un kmole du i-ème produit de combustion final et de la j-ème substance de départ ; n i, n j, respectivement, le nombre de kmoles du i-ème produit de réaction et de la j-ème substance de départ dans l'équation de la réaction de combustion ; Teneur en C, H, S, W, respectivement, % en poids carbone, hydrogène, soufre et humidité dans la substance ; О somme d'oxygène et d'azote, % poids ; Q H i est la chaleur inférieure de combustion du ième composant combustible du mélange gazeux, kJ/kmol ; ϕ gi teneur du ième composant combustible dans le mélange gazeux, % vol. Le calcul de la chaleur de combustion des mélanges gaz-air est effectué selon la formule SM 1 Q H Q Hφ G, (1,33) 1 SM où Q H est la chaleur de combustion du mélange gaz-air, kJ/m 3, kJ/ kmol; Q N chaleur inférieure de combustion d'une substance combustible, kJ/m 3, kJ/kmol ; ϕ g concentration de carburant dans un mélange avec un comburant, % vol. 5

26 Le débit (intensité) spécifique du dégagement de chaleur pendant la combustion est égal à q Q Н υ М, (1,34) où q est l'intensité du dégagement de chaleur spécifique kW/m ; υ m taux d'épuisement massique, kg/(m s). Le taux de dégagement de chaleur pendant la combustion, la chaleur du feu, est égal à q Q Н υ М F, (1,35) où q n est l'intensité du dégagement de chaleur, kW ; F surface de combustion, m Exemples Exemple 1. Déterminez la chaleur inférieure de combustion de l'acide acétique si sa chaleur de formation est de 485,6 kJ/mol. Solution Pour calculer à l'aide de la formule (1.3), nous écrivons l'équation de combustion de l'acide acétique dans l'oxygène CH 3 COOH + O CO + HO ; (396,9 + 4,1 485,6) 79,6 3 Q kJ/mol 79,6 1 kJ/kmol. H Pour calculer la quantité de chaleur dégagée lors de la combustion de 1 kg de carburant, il faut diviser la valeur obtenue par son poids moléculaire (64) 3 79, 6 1 Q H 1384 kJ/kg. 64 Exemple Calculer la chaleur inférieure de combustion de la masse organique de la composition : C 6%, H - 8%, O 8%, S%. Solution Selon la formule de D.I. Mendeleïev (1,31) () 5, Q H 339,9 8 kJ/kg. Exemple 3. Déterminer le pouvoir calorifique inférieur d'un mélange gazeux composé de CH 4 4 %, C 4 H 1 %, O 15 %, HS 5 %, NH 3 1 %, CO 1 %. Solution Pour chaque composant combustible du mélange, en utilisant la formule (1.3), nous trouvons la chaleur de combustion (tableau 1.6). 6

27 Équation thermique pour la réaction de formation de carburant, 1-3 kJ/kmol CH 4 + O CO + H O 75 C 4 H 1 + 6,5O 4CO + 5H O 13,4 Q N Chaleur de combustion, 1-3 kJ/kmol T a b face ,9 + 4, 75 86,3 Q 4 396, 13,5 N 666,1 H S + 1,5O H O + SO 1,1 Q 4, + 97,5 1,1 338,6 N NH 3 + 0,75O 1,5H O + 0,5N 46,1 Q 1,5 4, 46,1 317, N En utilisant la formule ( 1.3), on détermine la chaleur de combustion du mélange gazeux Q N (86, 1) 1 178,5 1, 1 kJ/kmol. Pour déterminer la chaleur de combustion de 1 m 3 d'un mélange gazeux, il faut diviser la valeur obtenue par le volume occupé par 1 kmol de gaz dans des conditions standards (4,4 m 3) : 3 178,5 1 Q N 5776 kJ/m 3 4.4 P r i M e r 4. Calculer la chaleur de combustion de 1 m 3 d'un mélange stoechiométrique hexane-air. Solution Nous trouvons la composition stoechiométrique du mélange combustible en utilisant l'équation de la réaction de combustion C 6 H,5O + 9,5 3,76N 6CO + 7HO + 9,5 3,76N. Le volume total des composants ayant réagi (1 + 9,5 + 9,5 3,76) est pris à 1%, et la quantité de carburant (1 kmol) correspondra à la concentration stoechiométrique 1 1 φ G,%. 1+ 9,5 + 9,5 3,76 La chaleur de combustion de 1 m 3 d'hexane est déterminée par la formule (1.3) Q 6 396, 167, 399,6 kJ/mol, H 399,6 H 1 4,4 3 3 Q 1 16, kJ/m 3. 7

28 Le volume d'un kmole de gaz dans des conditions standard est égal à 4,4 m 3. La chaleur de combustion de 1 m 3 d'un mélange stoechiométrique hexane-air est déterminée par la formule (1.33) 3 16, 1, Q 355 kJ/ m 3. 1 EXEMPLE 5. Déterminer l'intensité du dégagement de chaleur dans un incendie de masse organique (composition dans l'exemple), si le taux de combustion est de 15 kg/(m s) et la zone d'incendie est de 15 m. Solution. Selon formule (1,35) : 3 q 646,5 kW 59,5 MW. P 1 Tâches d'essai 1. Déterminer le pouvoir calorifique inférieur de 1 m 3 d'éthane, de propane, de butane, de pentane et d'hexane. Tracez la dépendance de Qn sur le poids moléculaire du carburant. Chaleur de formation de substances inflammables : éthane 88,4 kJ/mol, propane 19,4 kJ/mol, butane 3,4 kJ/mol, pentane 184,4 kJ/mol, hexane 11, kJ/mol Calculer la chaleur de combustion de 1 m 3 acétylène-air mélange aux limites de concentration inférieure et supérieure d'inflammation, ainsi qu'à la concentration stoechiométrique. Les limites de concentration d'inflammabilité (FLC) de l'acétylène sont de -81 %. Remarque : Tracez un graphique de la dépendance du pouvoir calorifique inférieur sur la concentration de carburant dans l'air. Lors du calcul de la chaleur de combustion d'un mélange sur un VCPV, il faut prendre en compte que seule une partie du carburant est capable de s'oxyder complètement dans l'air, le reste du carburant n'entrera pas dans une réaction de combustion en raison d'un manque d'oxydant. 3. Déterminer le pouvoir calorifique inférieur de 1 kg de bois de composition C 49%, H 8%, O 43%. Quelle est l'intensité spécifique du dégagement de chaleur dans un incendie si le taux de combustion massique est de 1 kg/(m·s) ? 4. Pour les conditions du problème précédent, déterminez le changement de la chaleur de combustion et l'intensité spécifique du dégagement de chaleur avec une teneur en humidité du bois (supérieure à 1 %) à hauteur de 3, 5, 1 et 15 %. Le taux de combustion du bois humide diminuera en conséquence à 0,9, 8,6 et 0,5 kg/(m s). Tracez un graphique de la dépendance de Q n et q à la teneur en humidité du matériau combustible. Remarque : Pour résoudre le problème, il est nécessaire de recalculer la composition du bois en tenant compte de l'humidité afin que la teneur de tous les composants soit égale à 1 %. 8

29 5. Déterminer l'intensité du dégagement de chaleur, kW, lors de la combustion d'un mélange gazeux de composition : CO 15 %, C 4 H 8 4 %, O %, H 14 %, CO 11 %, si la vitesse d'échappement est de 8 m 3 / s Calcul de la température de combustion et d'explosion La température de combustion est la température des produits de combustion dans la zone de réaction chimique. C'est la température maximale de la zone de flamme. La température de combustion et d'explosion est déterminée à partir de l'équation du bilan thermique Q H n C i 1 p (v) i (TG T) Dans ce cas, la température de combustion adiabatique est la température de combustion réelle. (1.36) * QН TG T +, (1.37) C T Г T + pi Q C pi, (1.38) * où T Г et T Г sont respectivement les températures adiabatiques et réelles de combustion ; T - température initiale ; je volume du ième produit de combustion ; QH - chaleur inférieure de combustion d'une substance ; Q est la chaleur utilisée pour chauffer les produits de combustion ; C i est la capacité calorifique du i-ème produit de combustion à volume constant. Dans ce cas, Q Q Н (1 - η), (1.39) où η est la part de la perte de chaleur résultant du rayonnement énergétique, de la sous-combustion chimique et mécanique. Le calcul de la température de combustion selon la formule (1.37) ou (1.38) ne peut être effectué que par la méthode des approximations successives, puisque la capacité calorifique des gaz dépend de la température de combustion (tableau 1.7) 9

30 Paramètres définis 1 Volume et composition des produits de combustion Pouvoir calorifique inférieur ou quantité de chaleur utilisée pour chauffer les produits de combustion (en présence de perte de chaleur) 3 Valeur moyenne de l'enthalpie des produits de combustion 4 Température de combustion T 1 par enthalpie moyenne en utilisant le tableau 1a ou 1b , en se concentrant sur l'azote (la teneur la plus élevée dans les produits de combustion) 5 Contenu calorifique des produits de combustion avec une température T 1 (Tableau 1a, 1b app.) 6 Si Q< Q Н (), то T >T 1 (en i (1.) kmol/kmol, m 3 /kg Q ou Q Н (1,3) kJ/kmol, kJ/kg Note Tableau 1.7 QН () HCP (1,4) i 1 Q Hi i (1,41) H i est l'enthalpie du i-ème produit de combustion ; i - / volume du i-ème produit de combustion si Q > QН (), alors T< T 1) 7 Q по формуле (1.41) 8 Расчет проводим до получения неравенства Q < QН () < Q 9 Температура горения (Н())(1) T Q Q T T T Г 1 + (1.4) Q Q Температура взрыва, протекающего в изохорно-адиабатическом режиме (при постоянном объеме) рассчитывается по уравнению теплового баланса (1.36) по методике, приведенной в табл Отличие заключается в том, что при расчетах вместо средней энтальпии продуктов горения и их теплосодержания (пп. 3-7) используется значение внутренней энергии газов (табл. приложения). Внутренняя энергия газов U C v T, где С v теплоемкость при постоянном объеме, кдж/(моль К), кдж/(м 3 К). Действительная температура горения на пожаре для большинства газообразных, жидких и твердых веществ изменяется в достаточно узких пределах (13-18 К). В связи с этим расчетная оценка действительной температуры горения может быть значительно упрощена, если теплоемкость продуктов горения выбирать при температуре 15 К: 3

31 Qн TG T +, (1.43) * C * où C Pi est la capacité thermique du i-ième produit de combustion à 15 K (tableau 1.8). Substance kJ/(m 3 K) Pi i Capacité thermique Tableau 1,8 kJ/(mol K) CO,7 5, SO,8 51,7 1-3 HO (vapeur) 1,78 39, N 1 ,4 31, Air 1,44 3,6 1- 3 Exemples Exemple 1. Déterminer la température de combustion adiabatique de l'alcool éthylique dans l'air. Solution. Nous effectuons le calcul selon le schéma donné dans le tableau. Puisqu'il s'agit d'une substance individuelle inflammable, pour déterminer le volume et la composition des produits de combustion, nous écrivons l'équation de la réaction chimique de combustion C H 5 OH + 3O + 3 3,76 NCO + 3HO + 3 3,76N. Par conséquent, les produits de combustion sont constitués de : CO mol, HO 3 mol, N 11,8 mol, 16,8 mol.. Nous déterminons la chaleur inférieure de combustion à l'aide de la formule (1.3). Dans le tableau 3 de l'annexe, nous trouvons la chaleur de formation du carburant - 78, kJ/mol Q H 396, - 78, 14, kJ/mol. 3. Enthalpie moyenne des produits de combustion 14, H moyenne 76,3 kJ/mol. 16,8 4. Puisque H moy est exprimé en kJ/mol, selon le tableau. 1a de l'application, en nous concentrant sur l'azote, nous sélectionnons la première température de combustion approximative T 1 1 o C. 5. Nous calculons le contenu calorifique des produits de combustion à 1 o C en utilisant la formule (1.41) Q 114,7 + 93,4 11,8 133,7 kJ/ mol. 31

32 6. Comparez QH et Q, puisque Q > QH, choisissez la température de combustion égale à environ C. 7. Calculez le contenu calorifique des produits de combustion à environ C : Q 18,6 + 88,1,8 11,8 135 kJ/mol. 8. Depuis Q< Q < Q, определим температуру горения по формуле (1.4) Н (14, 135)(1) T + 1 о С. 133,7 135 Г П р и м е р. Определить адиабатическую температуру горения органической массы, состоящей из С 6 %, Н 7 %, О 5 %, W 8 %. Р е ш е н и е. 1. Так как горючее представляет собой сложное вещество, состав продуктов горения рассчитываем по формулам () 6 CO 1,86 1,1 м /кг; 1,4, 88 H O 11, 1 + м 3 /кг; N ,1 м 3 /кг. 1 8 Общий объем продуктов горения равен 7, 1 м3 /кг.. Определим низшую теплоту cгорания вещества по формуле Д.И. Менделеева (1.31) Q Н 339,9 5-5,1() 3958,4 кдж/кг. 3. Определим среднюю энтальпию продуктов горения 3958,4 H CP 3417,7 кдж/м 3. 7,1 4. Так как величина энтальпии рассчитана в кдж/м 3, первую приближенную температуру выбираем по табл. 1б приложения. Ориентируясь на азот, принимаем Т 1 1 о С. 5. Рассчитываем теплосодержание продуктов горения при 1 о С по формуле (1.41) Q 5118, 1,1,9 5,1 5144,5 кдж/кг 6. Из сравнения Q Н и Q Q Н >Q sélectionner la deuxième température approximative égale à 19 o C. 7. Calculer le contenu calorifique des produits de combustion à 19 o C 3

33Q 4579,7 1,5, 5,1 498,8 kJ/kg. 8. Depuis Q< QН < Q, определим температуру горения (3958,4 498,8)(1 19) T Г о С,8 П р и м е р 3. Рассчитать действительную температуру горения фенола (H обр 4, кдж/моль), если потери тепла излучением составили 5 % от Q н, а коэффициент избытка воздуха при горении,. Р е ш е н и е. 1. Определим состав продуктов горения: C 6 H 5 OH + 7O + 7 3,76N 6CO + 3H O + 7 3,76N, 6 моль; 3 моля; 6, 3 моля, CO H O (,76)(, 1) 39, 98 N В моля, 75, 3 моля.. Определим низшую теплоту сгорания фенола (формула 1.3): Q Н 7 396, - 1 4, 35,7 кдж/моль, так как по условию задачи 5 % тепла теряется, определим количество тепла, пошедшее на нагрев продуктов горения (теплосодержание продуктов горения при температуре горения) (формула 1.39) Q 35,7(1 -,5) 65,5 кдж/моль. По формуле (1.43) определим действительную температуру горения 65,5 Т Г К. 3 1 (5,81 6,3+ 3,6 39,98) П р и м е р 4. Рассчитать температуру взрыва метановоздушной смеси стехиометрического состава. Р е ш е н и е. Расчет проводим по схеме, представленной в табл Объем и состав продуктов горения СН 4 + О + 3,76N СО + Н О + 3,76N. Продукты горения: CO 1 кмоль/кмоль, H O моль/моль, N 3,76 7,5 кмоль/кмоль.. Низшая теплота сгорания: Q Н 1 396,6 + 4, кдж/моль. 3. Средняя внутренняя энергия продуктов горения QН 86 U ср 76,8 кдж/моль. 1,5 33

34 4. Selon le tableau. Dans l'application, nous acceptons la première température approximative d'explosion (pour l'azote) T 1 7 o C. 5. Nous calculons l'énergie interne des produits de combustion à T 1 : U 1 pgi U i 1 18,9 + 1,4 + 7,5 7. 86, kJ/mole. 6. La comparaison des valeurs de Q H et U 1 montre que T 1 est surestimé. 7. Choisissez T 5 o C. U 1 118,3 + 94,3 + 7,5 64,3 789, kJ/mol. 8. Puisque U 1 > Q Н > U Т adulte, (7 5) 54 о С. 86, 789, Problèmes de test 1. Déterminer comment la température de combustion adiabatique change dans la série homologue d'hydrocarbures saturés (en utilisant l'exemple du méthane, propane, pentane et heptane). Dessinez un graphique de la dépendance de la température de combustion sur le poids moléculaire de la substance combustible. Déterminez comment la température de combustion adiabatique du bois de composition : C 49%, H 8%, O 43% change si la teneur en humidité (supérieure à 1 %) est de 5, 15 %. Dessinez un graphique de la température de combustion en fonction de la teneur en humidité du carburant. Remarque : Lors de la résolution du problème, il est nécessaire de recalculer la composition du bois afin que la quantité de tous les composants (y compris l'eau) soit de 1 %. 3. Déterminez comment la température de combustion adiabatique du benzène changera dans l'air et dans un environnement oxydant contenant 5, 3 et 4 % d'oxygène. Dessinez un graphique de la température de combustion en fonction de la teneur en oxygène. 4. Calculer la température réelle de combustion d'un mélange gazeux composé de 45 % H, 3 % C 3 H 8, 15 % O, 1 % N, si la perte thermique était de 3 % Q H, et le coefficient d'excès d'air lors de la combustion est 1,8. 5. Déterminer la quantité d'anthracite brûlé (C 1%) dans une pièce d'un volume de 18 m 3, si la température moyenne du volume est passée de 35 à 65 K. 6. Calculer la température réelle de combustion d'un mélange butane-air de concentration stoechiométrique à la limite inférieure de concentration d'inflammation (1,9 % de butane et 98,1 % d'air), si la perte de chaleur par rayonnement s'élève à % du pouvoir calorifique inférieur. 34

35 7. Déterminez comment la température de combustion de l'acétylène changera lorsqu'il sera dilué avec de l'azote à raison de 1,3 %, si la perte de chaleur par rayonnement est de 5 % du pouvoir calorifique inférieur, le coefficient d'excès d'air est de 1. Construisez un graphique de la température en fonction de la teneur en azote de l’acétylène. 8. Déterminez la durée de combustion du toluène à laquelle la température dans une pièce d'un volume de 4 m 3 augmentera de 95 à 375 K si son taux de combustion est de 15 kg/(m s) et que la zone d'incendie est de 5 m. , négliger l'augmentation du volume des produits de combustion au-dessus de l'air consommé. Devoir : Calculez la température de combustion de la ième substance (tableau 1.9). Numéro d'option Substance combustible Formule chimique Composition du milieu comburant 1 Mélange gazeux CO 4%, C 3 H 8 5%, CO 1% Air Tableau 1.9 Conditions de combustion α 1,4 η.5 Substance C 8%, H 5%, α 1, 6 composition complexe S 6%, W 9% - «- η,3 3 Acide propionique C 3 H 6 O O 5%, N 75% α 1,3 η,4 4 Glycérol C 3 H 8 O 3 Air α 1, η ,35 5 Éther d'acétate de butyle C 6 H 1 O - "- α 1,4 η,15 6 Éthylbenzène C 8 H 1 - "- α 1,5 η, 7 Substance de composition complexe C 8%, H 8%, O 5%, W 5% - «- α 1, η,35 8 Mélange de gaz CO 6%, H 4% - «- α 1,8 η,4 9 Ammoniac NH 3 - «- α 1, η , 1 Hexane C 6 H 14 - "- α 1,4 η,15 11 Nitroéthane C H 5 NO - "- α 1,5 η, 1 Alcool hexylique C 6 H 14 O Air α, η,1 35

36 Numéro d'option Fin du tableau. 1.9 Substance combustible Formule chimique Composition du milieu comburant Conditions de combustion C 75%, H 8%, - "- α 1, composition complexe C 1%, W 5% η,4 14 Substance 15 Mélange de gaz CH 4 7%, NH 3%, O 1% 16 Acide formique 17 Substance de composition complexe 18 Substance de composition complexe CH O O 5%, N 75% C 56%, H 14%, O%, W 1% C 78%, H 1%, O 1% 19 Mélange de gaz CO 75%, CH 4 5% Mélange de gaz C 3 H 8 7%, C 4 H 1%, O 1% C 85%, H 1%, O 5% 1 Substance de composition complexe Mélange de gaz C H 6 75%, CH 4 %, O 5% 3 Substance de composition complexe C 7%, H 16%, O 14% 4 Mélange de gaz CO 5%, CH 4 3%, CO % 5 Substance de composition complexe C 77%, H 13%, N 4%, O 6% - "- α 1,8 η, α, η.3 Air α 1, η.4 - "- α 1,6 η.15 - "- α 1,9 η, - "- α 1,8 η, - "- α 1,4 η,3 - "- α 1,7 η, - "- α 1, η,35 - "- α 1,9 η,15 - "- α 1, η,45 6 Éthylène C H 4 O 3% N 7% α 1,5 η.4 7 Alcool amylique C 5 H 1 O Air α, η.15 36

Chapitre 37 Limites de concentration de propagation de la flamme (allumage) La limite de concentration inférieure (supérieure) de propagation de la flamme est la concentration minimale (maximale) de carburant dans le comburant qui peut s'enflammer à partir d'une source d'énergie élevée avec propagation ultérieure de la combustion à l'ensemble du mélange. Formules de calcul La limite inférieure de concentration d'inflammation ϕ N est déterminée par la chaleur maximale de combustion. Il a été établi que 1 m 3 de divers mélanges gaz-air au NKPV émet une quantité moyenne constante de chaleur lors de la combustion de 183 kJ, appelée chaleur ultime de combustion. Par conséquent, φ Q 1 PR N, (.1) QН si l'on prend la valeur moyenne de Q PR égale à 183 kJ/m 3, alors φ N sera égal à φ N Q où Q N est la chaleur de combustion inférieure du combustible substance, kJ/m 3. Les CPV inférieur et supérieur peuvent être déterminés par la formule d'approximation Н ϕ () 1 Н В, (.) an + b où n est le coefficient stoechiométrique de l'oxygène dans l'équation de la réaction chimique ; a et b sont des constantes empiriques dont les valeurs sont données dans le tableau 1. Tableau a.1 Limites de concentration Valeurs du coefficient d'inflammabilité a b Inférieur 8,684 4,679 Supérieur n 7,5 1,55,56 n > 7,5,768 6,554 37

38 Les limites de concentration d'inflammation des vapeurs de substances liquides et solides peuvent être calculées si les limites de température φ Н (В) рн(В) 1, (.3) p où p Н (В) pression de vapeur saturée de la substance à une température correspondant à la limite inférieure (supérieure) d'inflammation, Pa ; p - pression ambiante, Pa. La pression de vapeur saturée peut être déterminée à partir de l'équation d'Antoine ou du tableau. 4 annexes B lg P A, (.4) C + t où A, B, C sont des constantes d'Antoine (tableau 1 annexe) ; t - température, C (limites de température). Pour calculer les limites de concentration d'inflammation des mélanges de gaz combustibles, utilisez la règle de Le Chatelier où φ Р 1 n() SM 1 φ N(B), (.5) μ i φ Н(В) i В inférieur (supérieur) CPV du mélange gazeux, % environ.; ϕ Н(в)i - Н(В) Р limite inférieure (supérieure) d'inflammabilité du i-ième gaz combustible %, vol. ; µ i est la fraction molaire du i-ième gaz combustible dans le mélange. Il faut garder à l’esprit que Σµ i 1, c’est-à-dire la concentration de composants inflammables du mélange gazeux est prise à 1 %. Si les limites de concentration d'inflammation à la température T 1 sont connues, alors à la température T elles sont calculées selon les formules où NG φ 1 NG φ T T T 1 φng NG 1 1 G T1 T VG φvg 1+ T T 1 φ 1 G T1 ; (.6), (.7) φ, limite inférieure de concentration d'inflammation, respectivement, aux températures T et T 1 ; φ VG et 1 φ VG sont respectivement la limite supérieure de concentration d'inflammation aux températures T 1 et T ; T G est la température de combustion du mélange. 38

39 Approximativement, lors de la détermination du LCPV, TG est considéré comme étant de 155 K, lors de la détermination du VCPV, de 11 K. Lorsque le mélange gaz-air est dilué avec des gaz inertes (N, CO, vapeur de H O, etc.), la région d'allumage se rétrécit : la limite supérieure diminue et la limite inférieure augmente. La concentration d'un gaz inerte (agent flegmatisant), à laquelle les limites inférieure et supérieure d'inflammabilité se chevauchent, est appelée concentration flegmatisante minimale ϕ f. La teneur en oxygène dans un tel système est appelée teneur minimale en oxygène explosif MVSC O) la teneur en oxygène inférieure au MVSC est appelée sûre. Les paramètres spécifiés sont effectués selon les formules φ Ф h + + f H f hф hi mi h 1 + h m Ф φ О О sans i i (φ. Certains φ О sans. Calcul 1 ; (.8) 1 φф ; (.9) 4,844 φ,φ 4, (.1) 1 O où ΔH f est la chaleur standard de formation de carburant, J/mol ; h i, h" i, h Ф constantes , selon le type d'élément chimique dans la molécule de carburant et le type de flegmatiseur (tableau 11 de l'annexe) ; m i est le nombre d'atomes du i-ème élément (groupe structural) de la molécule de combustible. Le calcul de ces paramètres peut être effectué par une autre méthode physiquement plus transparente en résolvant l'équation du bilan thermique ( 1.36) dans les deux conditions suivantes : - au point de flegmatisation, le le mélange combustible a une température maximale de combustion de 15 K ; - le mélange est stoechiométrique lors de l'oxydation du carbone en CO, de l'hydrogène en H O. L'équation du bilan thermique (1.36) dans le cas d'une dilution avec un gaz neutre se présente sous la forme : Q Н (Т) Г Т С ni + Срф nф Рi, (.11) * où Т Г la température maximale de combustion est de 15 K ; C Pi, C Рf, respectivement, la capacité calorifique du i-ième produit de combustion et du gaz neutre (phlegmatiseur), kJ/(mol K) ; n i est le nombre de moles du i-ème produit de combustion du mélange stoechiométrique, mol/mol ; n f nombre de moles de gaz neutre au point de flegmatisation, mol/mol. 39

40 De (.11) n Q (T G T) CPi C (T T) N f RF G n i (.1) En prenant le volume de tous les composants du mélange gaz-air comme 1 %, la concentration (% vol.) de chacun parmi eux n i φi est déterminé 1 (.13) n + n + n + n G O N f Exemples Exemple 1. En utilisant la chaleur maximale de combustion, déterminez la limite inférieure de concentration d'inflammation du butane dans l'air. Solution Pour le calcul en utilisant la formule (.1) dans le tableau. Dans 3 applications, nous trouvons que la chaleur de combustion la plus faible de la substance est de 88,3 kJ/mol. Cette valeur doit être convertie en une autre dimension kJ/m 3 : 88, kJ/m 3., 4 À l'aide de la formule (.1), nous déterminons le LCPV φ H 1,4 %. 18,7 13 Selon tableau. En se reportant à la figure 4 de l'annexe on constate que la valeur expérimentale de ϕ H est de 1,9 %. L'erreur relative de calcul était donc de 1,9 1,4 N 1 5 %. 1.9 Exemple Déterminer les limites de concentration d'inflammabilité de l'éthylène dans l'air. Solution Nous calculons le CPV en utilisant la formule d’approximation. On détermine la valeur du coefficient stoechiométrique pour l'oxygène. Ainsi, n 3, puis CH 4 + 3O CO + H O. 1 φ H 3,5 8,679% ; 1 φ 18, 1,55 3,56 3 V + %. Déterminons l'erreur relative de calcul. D'après le tableau 4 applications, les valeurs limites expérimentales sont 3, 3, : 4


MINISTÈRE DE L'ÉDUCATION ET DES SCIENCES DE LA FÉDÉRATION DE RUSSIE Université technologique d'État de Belgorod nommée d'après. V.G. Choukhova THÉORIE DE LA COMBUSTION ET DE L'EXPLOSION Instructions méthodologiques pour la conduite des cours pratiques

Thème 4 « INCENDIE ET ​​EXPLOSION MÉLANGES DANGEREUX DE VAPEUR ET GAZ-AIR » Leçon 4.2 « Calcul des limites d'inflammabilité de concentration » (2 heures) Formules de calcul Limite inférieure d'inflammabilité de concentration (LCFL) n

CALCULS THERMIQUES DANS LES PROCÉDÉS DE COMBUSTION 1. PROPRIÉTÉS DES SUBSTANCES COMBUSTIBLES Selon la composition des phases, les substances combustibles peuvent être liquides, solides et gazeuses. Chaleur de combustion (pouvoir calorifique) du carburant Q

Travaux d'essai dans la discipline « Théorie de la combustion et des explosions » Option 1 (kJ/kg) d'un composé toluène individuel (C 6 H 5 CH 3). 2. Déterminer le volume d'air nécessaire à la combustion de 1 kg de benzène (l)

ÉMERCOM DE RUSSIE ÉTABLISSEMENT D'ENSEIGNEMENT FÉDÉRAL D'ÉTAT D'ENSEIGNEMENT PROFESSIONNEL SUPÉRIEUR UNIVERSITÉ D'ÉTAT D'INCENDIE DE SAINT-PÉTERSBOURG ÉMERCOM DE RUSSIE CHIMIE DES PROCÉDÉS

MINISTÈRE DE L'ÉDUCATION ET DES SCIENCES DE LA RF Établissement d'enseignement budgétaire d'État d'enseignement professionnel supérieur "UNIVERSITÉ D'ÉTAT DE PENZA" Atelier THÉORIE DE LA COMBUSTION ET DE L'EXPLOSION

AGENCE FÉDÉRALE DES TRANSPORTS FERROVIAIRES Université d'État des transports de l'Oural Département de sécurité de la technosphère A. Zh. Khvorenkova THÉORIE DE LA COMBUSTION ET DE L'EXPLOSION Ensemble des tâches sur le terrain

MINISTÈRE DE LA FÉDÉRATION DE RUSSIE POUR LA DÉFENSE CIVILE, LES SITUATIONS D'URGENCE ET L'ÉLIMINATION DES CATASTROPHES Académie des pompiers de l'État I. R. Begishev THÉORIE DE LA COMBUSTION

MINISTÈRE DE L'ÉDUCATION ET DES SCIENCES DE L'Établissement d'enseignement budgétaire de l'État fédéral russe d'enseignement professionnel supérieur « Université technique d'État d'Omsk » FONDEMENTS PHYSIQUES ET CHIMIQUES DU DÉVELOPPEMENT

MINISTÈRE DE LA FÉDÉRATION DE RUSSIE POUR LA DÉFENSE CIVILE, LES SITUATIONS D'URGENCE ET L'ÉLIMINATION DES CATASTROPHES Académie des pompiers de l'État COURS I. R. Begishev

Combustible pour fours tubulaires. Calcul du processus de combustion du carburant Informations générales sur le carburant Le carburant est une substance organique brûlée pour produire de la chaleur. Les principaux composants combustibles du carburant sont

Yu.S. Biryulin, V.N. Mikhalkin CALCUL THERMODYNAMIQUE DE LA CHALEUR DE COMBUSTION DES HYDROCARBURES La chaleur de combustion est importante pour évaluer le risque d'incendie des substances et constitue également un indicateur de la pratique

Thème 2 « BILAN MATÉRIEL ET THERMIQUE DES PROCÉDÉS DE COMBUSTION ET D'EXPLOSION » Leçon 2.2 « Bilan thermique des processus de combustion » 1 Questions abordées : 1. Chaleur de combustion. 2. Température de combustion. Littérature : 1.

O. V. Arkhangelskaya, I. A. Tulkov, Université d'État de Moscou. M. V. Lomonosova Une tâche difficile ? Commençons dans l'ordre. Comme le montre la pratique, la thermochimie est l'une des sections de la chimie les plus difficiles pour les candidats. Résoudre des problèmes

17. Régularités des processus chimiques. Le concept de la vitesse d'une réaction chimique. Facteurs influençant la modification de la vitesse d'une réaction chimique La vitesse d'une réaction chimique est le rapport de la variation de la concentration

Agence fédérale des transports ferroviaires Université d'État des transports de l'Oural Département de la sécurité des personnes N.V. Gushchina THÉORIE DE L'INCLUSION ET DE L'EXPLOSION Ekaterinbourg 11 Fédéral

MINISTÈRE DE L'ÉDUCATION ET DES SCIENCES DE LA FÉDÉRATION DE RUSSIE Établissement d'enseignement budgétaire de l'État fédéral d'enseignement professionnel supérieur « RECHERCHE NATIONALE TOMSK POLYTECHNIC

AGENCE FÉDÉRALE POUR L'ÉDUCATION Établissement d'enseignement budgétaire de l'État d'enseignement professionnel supérieur "UNIVERSITÉ POLYTECHNIQUE NATIONALE DE RECHERCHE DE TOMSK" Module

Chaleur de combustion et température de combustion du carburant Leçon 3 Carburants Le carburant est une source d'énergie ; substance inflammable qui produit une quantité importante de chaleur lors de la combustion Combustible solide : naturel

MINISTÈRE DE L'ÉDUCATION ET DES SCIENCES DE LA FÉDÉRATION DE RUSSIE Université technique d'État de Briansk APPROUVÉ par le recteur de l'université O.N. Fedonin 2014 FOURNES DE FONDERIE CALCUL DES CARACTÉRISTIQUES DE COMBUSTION

EMERCOM DE RUSSIE SAINT-PÉTERSBOURG UNIVERSITÉ DES POMPIERS D'ÉTAT Korobeynikova E.G. THÉORIE DE LA COMBUSTION ET DE L'EXPLOSION Manuel méthodologique pour la réalisation du test pour les étudiants par correspondance

Thème 5 Déterminer les conditions de sécurité d'utilisation des bouteilles de gaz inflammables Objectif : acquérir des compétences pratiques pour effectuer des calculs techniques pour évaluer les conditions de sécurité d'utilisation des bouteilles

MINISTÈRE DE LA FÉDÉRATION DE RUSSIE POUR LA DÉFENSE CIVILE, LES SITUATIONS D'URGENCE ET L'ÉLIMINATION DES CATASTROPHES Académie des pompiers de l'État THÉORIE DE L'INCLUSION ET DE L'EXPLOSION

Option 1 1 2 3 4 Carbure de calcium acétylène benzène nitrobenzène aniline 5 6 éthylène éthanol 3. Quel volume d'air est consommé pour brûler 25 litres de méthylamine contenant 4 % d'impuretés ininflammables ? Fraction volumique d'oxygène

UDC 64.84.4 I.O. Stoyanovitch, V.S. Saushev, Le Xuan Ty (Russie, Vietnam) MÉTHODES DE CALCUL POUR DÉTERMINER LA TEMPÉRATURE D'ÉCLAIR DE LIQUIDES INDIVIDUELS DANS UNE CRUCICE FERMÉE Le champ d'application et les définitions sont présentés

Travaux de laboratoire « Calcul du mode de transformation explosive du mélange air-carburant » Algorithme de calcul. La procédure de réalisation du calcul est déterminée conformément à la méthodologie RD 03-40901 « Méthodologie d'évaluation des conséquences

1. Fraction massique d'un élément dans une substance. La fraction massique d'un élément est sa teneur dans une substance en pourcentage en masse. Par exemple, une substance de composition C 2 H 4 contient 2 atomes de carbone et 4 atomes d'hydrogène. Si

FACULTÉ UNIVERSITAIRE AGRICOLE D'ÉTAT DE NOVOSIBIRSK D'AGRONOMIE THÉORIE DE LA COMBUSTION ET DE L'EXPLOSION Collection de tâches et d'exercices pour effectuer des tests NOVOSIBIRSK 215 1 UDC 544.45 (75.) BBK 24.46,

UNIVERSITÉ TECHNIQUE OUVERTE DES COMMUNICATIONS D'ÉTAT DE RUSSIE DU MINISTÈRE DES COMMUNICATIONS DE LA FÉDÉRATION DE RUSSIE 16/18/2 Agréée par le département « Génie thermique et hydraulique dans le transport ferroviaire »

MINISTÈRE DE L'ÉDUCATION ET DES SCIENCES DE L'Établissement d'enseignement budgétaire de l'État fédéral russe d'enseignement professionnel supérieur « Université technique d'État d'Ukhta » (USTU) CALCUL DES LIMITES DE RISQUE

Thème 4 Sécurité contre les explosions et les incendies en production Objectif : acquérir des compétences pratiques dans la réalisation de calculs techniques pour évaluer les risques d'explosion et d'incendie des entreprises industrielles. Plan

Place de la discipline dans la structure du programme éducatif La discipline « Théorie de la combustion et de l'explosion » est une discipline de la partie de base. Le programme de travail est établi conformément aux exigences de l'État fédéral

CHIMIE GÉNÉRALE, INORGANIQUE ET PHYSIQUE TÂCHES INDIVIDUELLES Tâche 1 Calculer la quantité de chaleur nécessaire pour chauffer n moles de substance A d'une température de 298 K à une température T à constante

Conférence 9 13. 4. 6 7.8. Calcul de la constante d'équilibre via la fonction de partition moléculaire Z. 7.9. Calcul de l'équilibre de systèmes chimiques complexes. Problème de cours At P atm et T98 K pour la réaction gazeuse 1 SO + 5O

Ministère de l'Éducation et des Sciences de la Fédération de Russie Université d'État de l'Oural du Sud Département de la sécurité des personnes 6 (7) B16 M.Yu. Babkin, S.I. Borovik THÉORIE DE LA COMBUSTION ET DE L'EXPLOSION Éducatif

Planification thématique en chimie pour l'année académique 2017-2018, 9e année Manuel : O.S. GABRIELYAN. CHIMIE. 8ÈME ANNÉE. M., "DROFA", 2007-2012. Contenu du matériel pédagogique Dates Minimum obligatoire INTRODUCTION.

MINISTÈRE DE L'ÉDUCATION ET DES SCIENCES DE LA FÉDÉRATION DE RUSSIE UNIVERSITÉ D'ÉTAT D'ARCHITECTURE ET D'INGÉNIERIE DE NOVOSIBIRSK (SIBSTRIN) Département de Chimie CINÉTIQUE CHIMIQUE ET ÉQUILIBRE Missions individuelles

VÉRIFIER LE TRAVAIL Thème : « Alcools monohydriques » 1 1. RAPPELER LES PROPRIÉTÉS CHIMIQUES ET LA PRÉPARATION DES ALCOOLS MONO ACHOLOGIQUES. 2. EFFECTUEZ LES TESTS PROPOSÉS 22 et 23 (VOTRE CHOIX) PROPRIÉTÉS CHIMIQUES DES ALCOOLS MONO-ACHOLOGIQUES

2.1. Masse des atomes et des molécules Pour mesurer les masses des atomes et des molécules en physique et en chimie, un système de mesure unifié a été adopté. Ces quantités sont mesurées en unités relatives – unités de masse atomique. Unité atomique

Test Éléments contrôlés des tâches de connaissances 1-2 Classification des substances organiques 3 Groupes fonctionnels des principales classes de composés organiques 4 Homologues et leurs noms 5 Isomères et leurs noms

MINISTÈRE DE L'ÉDUCATION ET DES SCIENCES DE LA FÉDÉRATION DE RUSSIE AGENCE FÉDÉRALE POUR L'ÉDUCATION INSTITUTION D'ENSEIGNEMENT D'ÉTAT D'ENSEIGNEMENT PROFESSIONNEL SUPÉRIEUR « UNIVERSITÉ CIVILE D'ÉTAT DE ROSTO »

Test final (1er semestre), option 1 1. Pour la réaction 2 HCl (g) = H 2 + Cl 2, calculer : K p, K s, K P, 625 si les données suivantes sont connues : H 289 U, H 625 , A , HCl (g) Cl 2 (g) H 2 (g) H arr.

GÉORGIE. Tikhanovskaya L.M. Voropai BASES PHYSIQUES ET CHIMIQUES DU DÉVELOPPEMENT ET DE LA LUTTE CONTRE LES INCENDIES Vologda 2014 Ministère de l'Éducation et des Sciences de la Fédération de Russie Université d'État de Vologda G.A. Tikhanovskaïa

Planification thématique en chimie (externat) pour l'année académique 2016-2017 en 11e année Manuel : O.S. GABRIELYAN. CHIMIE. 11e ANNÉE. UN NIVEAU DE BASE DE. M., "DROFA", 2007-2015. Contenu semestriel du matériel pédagogique

Agence fédérale pour l'éducation Établissement d'enseignement fédéral d'enseignement professionnel supérieur Université d'État de Novgorod nommée en l'honneur de la Faculté de Yaroslav le Sage

CALENDRIER ET PLANIFICATION THÉMATIQUE POUR LA CHIMIE EN 10E ANNÉE SCOLAIRE 2009-2010. 2 heures par semaine. Programme pour les écoles secondaires, gymnases, lycées. Chimie 8-11 années, M. "Drofa", 2009. Manuel de base :

Agence fédérale pour l'éducation Établissement d'enseignement public d'enseignement professionnel supérieur Université d'État de Novgorod nommée d'après. Yaroslav le Sage Faculté des Sciences Naturelles

TRAVAIL DE DIAGNOSTIC en CHIMIE 10e année 6 avril 2011 Option 1 A1. Les substances organiques comprennent a) C 2 H 2 b) CaCO 3 c) C 2 H 5 OH d) CO e) C 2 H 5 NH 2 1) a, b, d 2) a, c, e 3) b, c , ré 4) b,

TÂCHES pour l'étape 2 de l'Olympiade « Premiers pas en médecine » en chimie Nom complet CLASSE ADRESSE DE L'ÉCOLE, TÉLÉPHONE Option 1 (60 points) PARTIE 1 (12 points) Lors de l'accomplissement des tâches de cette partie dans le formulaire de réponse 1 sous le numéro

I. A. Gromchenko Recueil de problèmes de chimie pour la 8e année Centre d'éducation de Moscou 109 2009 1. Fraction massique d'un élément. Calculs à l'aide de formules. 1.1. Quelle substance a une molécule plus lourde : BaO, P 2 O 5, Fe 2 O 3 ? 1.2.

Test final de chimie pour la 10e année, année académique 2017-2018 Option 1. Partie A. Lors de l'exécution des tâches de cette partie (A1-A15), parmi les quatre options proposées, choisissez la bonne. Sur le formulaire

MINISTÈRE DE L'ÉDUCATION ET DES SCIENCES DE LA FÉDÉRATION DE RUSSIE Établissement d'enseignement budgétaire de l'État fédéral d'enseignement professionnel supérieur "UFA STATE AVIATION TECHNICAL

OPTION 1 1. Deux récipients d'une capacité de 0,2 et 0,1 litre sont séparés par un piston mobile qui ne conduit pas la chaleur. La température initiale du gaz dans les cuves est de 300 K, la pression est de 1,01 10 5 Pa. Le plus petit récipient a été refroidi à 273 K et le plus grand

1. La charge du noyau d'un atome de fer est : 1) +8 ; 2) +56 ; 3) +26 ; 4) +16. Version de démonstration du travail pour le test d'entrée en chimie Partie 1 2. Dans quelle rangée se trouvent les formules de substances avec uniquement des substances covalentes

Billets d'examen de chimie, 10e année Billet 1 1. Principes de base de la théorie de la structure chimique des substances organiques A.M. Butlerov. Structure chimique comme ordre de connexion et influence mutuelle des atomes

Version démo du travail de chimie pour le cours de 0e partie A.. ​​​​Lorsque vous effectuez la tâche dans la liste de réponses proposée, sélectionnez deux bonnes et notez les numéros sous lesquels elles sont indiquées. Pour l'éthanol, les éléments suivants sont vrais :

Afin de simplifier le calcul, toutes les substances inflammables sont divisées en trois types : mélanges individuels, complexes, de gaz inflammables (tableau 1.2.1).

Tableau 1.2.1

Type de substance inflammable

Formules de calcul

Dimension

Substance individuelle

(1.2.2)

Substance de composition complexe

(1.2.3)

(1.2.4)

(1.2.5)

Mélange de gaz

(1.2.7)


(1.2.8)

Ici
- volume théorique des produits de combustion ;
-quantité du ième produit de combustion dans l'équation de réaction, kmol ; - quantité de carburant, kmol ; - volume de 1 kmol de gaz ;
- poids moléculaire du carburant ;
-volume du produit de réaction ; C, H, S, O, N – teneur des éléments correspondants (carbone, hydrogène, soufre, oxygène et azote) dans la substance combustible, % en poids ; - teneur du ième composant combustible dans le mélange gazeux, % vol. ;
- contenu jeème composant ininflammable dans le mélange gazeux, % vol.

Le volume pratique (total) des produits de combustion comprend le volume théorique des produits de combustion et l'excès d'air.

(1.2.9)


(1.2.10)

Composition des produits de combustion, c'est-à-dire le contenu du ième composant est déterminé par la formule

(1.2.11)


- contenu je- ème composant dans les produits de combustion, % vol.;

- volume je- le composant, m 3, kmol ;

- volume total de produits de combustion, m 3, kmol.

Lors de la combustion dans un excès d'air, les produits de combustion contiennent de l'oxygène et de l'azote.

(1.2.12)

(1.2.13)

-volume théorique d'azote dans les produits de combustion, m 3, kmol.

(1.2.14)

Exemples

Exemple 1. Quelle quantité de produits de combustion sera libérée lors de la combustion de 1 m 3 d'acétylène dans l'air si la température de combustion est de 1450 K.

Composé chimique combustible-individuel (formule 1.2.1). Écrivons l'équation de la réaction chimique de combustion

C2H2+
O2+
N 2 =2CO 2 +H 2 O+
N 2

Volume de produits de combustion dans des conditions normales

m 3 /m 3

Volume des produits de combustion à 1450 K


m 3 /m 3

Exemple 2. Déterminer le volume de produits de combustion lors de la combustion de 1 kg de phénol, si la température de combustion est de 1200 K, pression 95000 Pa, coefficient d'excès d'air 1,5.

Un composé chimique individuel inflammable (formule 1.2.2). Écrivons l'équation de la réaction chimique de combustion

C6H5OH+
O2+
N 2 =6CO 2 +3H 2 O+
N 2

Le poids moléculaire du carburant est de 98.

m 3 /kg

Volume d'air pratique dans des conditions normales

Volume de produits de combustion dans des conditions données

m 3 /m 3

Exemple 3. Déterminer le volume de produits de combustion lors de la combustion de 1 kg de masse organique de la composition : C-55%, O-13%, H-5%, S-7%, N-3%, W 17% , si la température de combustion est de 1170 K, coefficient d'excès d'air – 1,3.

Une substance inflammable de composition complexe (formules 1.2.3 - 1.2.6). Composition théorique des produits de combustion dans des conditions normales

m 3 /kg

m 3 /kg

Le volume théorique total des produits de combustion dans des conditions normales

=1+0,8+0,05+4,7=6,55 m 3 /kg

Volume pratique de produits de combustion dans des conditions normales

=6,55+0,269
(1,3-1)=6,55+1,8=8,35 m 3 /kg

Volume pratique de produits de combustion à température de combustion

=
m 3 /kg.

Exemple 4. Calculer le volume de produits de combustion lors de la combustion de 1 m 3 d'un mélange gazeux constitué de C 3 H 6 -70%, C 3 H 8 -10%, CO 2 -5%, O 2 -15%, si la température de combustion est de 1300 K, coefficient d'excès d'air – ​​2,8. Température ambiante 293 K.

Le carburant est un mélange de gaz (formule 1.2.7).

Le volume des produits de combustion est déterminé par la formule (1.2.8)

m 3 /m 3

m 3 /m 3

Étant donné que le mélange gazeux contient de l'oxygène, il oxydera certains des composants inflammables, par conséquent, le débit d'air diminuera (formule 1.1.5).

Dans ce cas, il est plus pratique de déterminer le volume théorique d'azote à l'aide de la formule (1.2.14)

m 3 /m 3

Volume théorique des produits de combustion

Volume pratique de produits de combustion

Volume de produits de combustion à une température de 1300 K

m 3 /m 3 .

Exemple 5. Déterminer la composition des produits de combustion de la méthyléthylcétone.

Avec cette formulation du problème, il est rationnel de déterminer directement à partir de l'équation de combustion le volume de produits en kmoles libérés lors de la combustion de 1 kmole de carburant

kmol;
kmol;
kmol;
kmol.

A l'aide de la formule (1.2.11) on trouve la composition des produits de combustion

Exemple 6. Déterminer le volume et la composition des produits de combustion de 1 kg d'huile minérale avec la composition : C-85%, H-15%, si la température de combustion est de 1450 K, le coefficient d'excès d'air est de 1,9.

Solution. À l'aide des formules (1.2.3 - 1.2.6), nous déterminons le volume des produits de combustion

m 3 /kg

m 3 /kg

m 3 /kg

Volume théorique des produits de combustion dans des conditions normales

Volume pratique de produits de combustion dans des conditions normales formule (1.2.10)

Volume de produits de combustion à une température de 1450 K

m 3 /kg

Évidemment, la composition des produits de combustion ne dépend pas de la température de combustion, il convient donc de la déterminer dans des conditions normales. D'après les formules (1.2.11;1.2.13)

;
;


Exemple 7. Déterminez la quantité d'acétone brûlée, en kg, si le volume de dioxyde de carbone libéré, ramené aux conditions normales, était de 50 m 3.

Écrivons l'équation de la réaction de combustion de l'acétone dans l'air

De l'équation, il résulte que lors de la combustion à partir de 58 kg (poids moléculaire de l'acétone)
m 3 dioxyde de carbone. Ensuite, pour former 50 m 3 de dioxyde de carbone, le Mg de carburant doit réagir

kg

Exemple 8. Déterminer la quantité de masse organique brûlée de composition C-58%, O-22%, H-8%, N-2%, W-10% dans une pièce d'un volume de 350 m 3, si le la teneur en dioxyde de carbone était de 5 %.

Solution. Déterminons le volume de dioxyde de carbone libéré

m3.

A l'aide de la formule (1.2.6) pour une substance de composition complexe, on détermine le volume de CO 2 dégagé lors de la combustion de 1 kg de carburant,

m 3 /kg.

Déterminons la quantité de substance brûlée

kg.

Exemple 9. Déterminer le moment où la teneur en dioxyde de carbone dans une pièce d'un volume de 480 m 3 résultant de la combustion du bois (C-45%, H-50%, O-42%, W-8%) était de 8 %, si le taux de combustion massique spécifique du bois est de 0,008 kg/(m 2 s) et que la surface de combustion est de 38 m 2. Lors de la résolution, ne tenez pas compte des échanges gazeux avec l'environnement et négligez la dilution résultant du dégagement de produits de combustion.

La dilution des produits de combustion n'étant pas prise en compte, on détermine le volume de dioxyde de carbone dégagé suite à la combustion, correspondant à 8 % de sa teneur dans l'atmosphère.

m3

À partir de l'expression (1.2.3), nous déterminons la quantité de matériau combustible qui doit brûler pour qu'un volume donné de dioxyde de carbone soit libéré.

kg.

Nous déterminons la durée de combustion en fonction du rapport


,

- le temps de combustion ;

Mg- masse de bois brûlé, kg ;

- taux massique de combustion du bois, kg/(m 2 s) ;

F- surface de combustion, m2 ;

min.

Mission de travail indépendante

Tâche 3 : Déterminer le volume de produits de combustion lors de la combustion de 1 kg d'une substance donnée, si la température de combustion... K, pression... mmHg, = ... .

Substance

T p. ex., K

R., mmHg.

Amylbenzène

Alcool N-amylique

Acétate de butyle

Alcool butylique

L'éther diéthylique

Èsprit blanc

Éthylène glycol

Alcool tert-amylique

Alcool méthylique

Amylméthylcétone

Butylbenzène

Éther butylvinylique

Éthanol

Alcool butylique

Tâche 4 : Déterminer le volume et la composition (% vol.) des produits de combustion libérés lors de la combustion de 1 m 3 de gaz combustible, si la température de combustion était... K, pression... mm Hg.

Substance

T p. ex., K

R., mmHg.

Acétylène

Monoxyde de carbone

Sulfure d'hydrogène

Acétylène

Monoxyde de carbone

Sulfure d'hydrogène

Monoxyde de carbone

Acétylène

Acétylène

Monoxyde de carbone